SlideShare a Scribd company logo
Photo: Scanning electron micrograph of the glomerulus in a human kidney.
From: Widmaier EP. Vander’s Human Physiology: The Mechanisms Of Body Function, 13th Ed. New York, NY: McGraw-Hill Companies, Inc., 2014: 490
Marc Imhotep Cray, M.D.
Learning Objectives:
1. List major types of diuretics and relate them to their sites of action.
2. List the major applications, toxicities, and the efficacy of thiazides, loop
diuretics and potassium-sparing diuretics.
3. Describe two drugs that reduce potassium loss during diuresis.
4. Describe a therapy that will reduce calcium excretion in patients who have
recurrent urinary stones.
5. Discuss the principle of force diuresis.
6. Describe drugs for reducing urine volume in nephrogenic diabetes insipidus.
7. Understand the usefulness of altering urine pH by drugs.
8. Discuss the mechanisms by which drugs and chemicals damage the kidney.
9. Understand how to select and prescribe drugs for patients with renal
impairment.
2
Companion: Renal Pharmacology eNotes
Marc Imhotep Cray, M.D.
Some Relevant Drugs:
3
A. Carbonic Anhydrase
Inhibitors
Acetazolamide
dichlorphenamide
methazolamide
dorzolamide
B. Osmotic Diuretics
mannitol
C. Loop Diuretics
furosemide
bumetanide
torsemide
ethacrynic acid
D. Thiazides & Thiazides-like
chlorthalidone
chlorothiazide
hydrochlorothiazide
metolazone
indapamide
E. Potassium-sparing
diuretics
spironolactone
eplerenone
triamterene
amiloride
F. ADH antagonists
demeclocycline
lithium
lixivaptan
tolvaptan
conivaptan
Marc Imhotep Cray, M.D.
Topical Outline:
4
o Role of Renal System
 Volume Homeostasis
o General Principles of Diuretic Action
o Individual Agents/Classes
 High-efficacy (loop) diuretics
 Moderate-efficacy diuretics
 Low-efficacy diuretics
 Osmotic diuretics
 Carbonic Anhydrase Inhibitors
o Adverse effects of diuretics and Drug-Drug Interactions
o Alteration of Urine pH
o Alkalinization
o Acidification
o Drugs and the Kidney and Prescribing in Renal Disease
o ADH Antagonists
 Clinical Cases and Discussions
 Practice MCQs
Marc Imhotep Cray, M.D.
Key Abbreviations
5
 PCT, Proximal convoluted tubule
 DCT, Distal convoluted tubule
 TAL, thick ascending limb of the loop of Henle
 CCD, cortical collecting duct (including late DCT forming initial
collecting duct)
 MCD, medullary collecting duct
 GFR, glomerular filtration rate
 ENaC, epithelial sodium channel
 NCC, Na-Cl cotransporter (formerly NCCT or thiazide-sensitive
Na–Cl co-transporter)
 NKCC2, Na–K–2Cl co-transporter
 ROMK, rectifying outer medullary potassium channel
Marc Imhotep Cray, M.D.
High-Yield Terms to Learn
6
 Bicarbonate diuretic A diuretic that selectively increases sodium
bicarbonate excretion. Example: a carbonic anhydrase inhibitor
 Diluting segment A segment of nephron that removes solute
without water; TAL and DCT are active salt-reabsorbing
segments that are not permeable by water
 Hyperchloremic metabolic acidosis A shift in body electrolyte
and pH balance involving elevated serum chloride, diminished
bicarbonate concentration, and a decrease in pH in the blood.
Typical result of bicarbonate diuresis
 Hypokalemic metabolic alkalosis A shift in body electrolyte
balance and pH involving a decrease in serum potassium and an
increase in blood pH. Typical result of loop and thiazide diuretic
actions
Marc Imhotep Cray, M.D.
High-Yield Terms to Learn cont.
7
 Nephrogenic diabetes insipidus Loss of urine-concentrating
ability in kidney caused by lack of responsiveness to ADH (ADH
is normal or high)
 Pituitary diabetes insipidus Loss of urine-concentrating ability
in kidney caused by lack of ADH (ADH is low or absent)
 Potassium-sparing diuretic A diuretic that reduces exchange of
potassium for sodium in collecting tubule; a drug that increases
sodium and reduces potassium excretion. Example: aldosterone
antagonists
 Uricosuric diuretic A diuretic that increases uric acid excretion, ,
usually by inhibiting uric acid reabsorption in the proximal
tubule. Example: ethacrynic acid
Marc Imhotep Cray, M.D.
Key Concepts in Clinical Renal Pharmacology
8
 Diuretic drugs: their sites and modes of action, classification, adverse
effects and uses in cardiac, hepatic, renal and other conditions.
 Carbonic anhydrase inhibitors.
 Cation-exchange resins and their uses.
 Alteration of urine pH.
 Drugs and the kidney.
 Adverse effects.
 Drug-induced renal disease: by direct and indirect biochemical effects and
by immunological effects.
 Prescribing for renal disease: adjusting the dose according to the
characteristics of the drug and to the degree of renal impairment.
 Nephrolithiasis and its management.
 Pharmacological aspects of micturition.
 Benign prostatic hyperplasia.
 Erectile dysfunction.
Marc Imhotep Cray, M.D.
Role of Renal System
9
 The kidneys comprise only 0.5% of body-weight, yet they
receive 25% of the cardiac output.
 Drugs that affect renal function have important roles in cardiac
failure and hypertension
 Disease of kidney must be taken into account when prescribing
drugs that are eliminated by it
 drugs can damage kidney and disease of kidney affects
responses to drugs (will be covered elsewhere)
Marc Imhotep Cray, M.D.
Role of Renal System (2):
Volume Homeostasis
10
Kidneys are part of an integrated homeostatic mechanism for
maintaining volume of extracellular fluid (ECF) and thus
mean arterial pressure (MAP)
Other organs involved in this mechanism include:
 Heart (eg, cardiac output and heart rate),
 CNS (eg, sympathetic tone and ADH release),
 Lungs (eg, conversion of angiotensin I to angiotensin II), and
 Adrenal gland (eg, release of aldosterone)
Marc Imhotep Cray, M.D.
Volume Homeostasis (2)
11
Several feedback control mechanisms operate among
components of this control mechanism ensure responses to
 volume expansion (increased extracellular fluid) and
 volume contraction (decreased extracellular fluid)
Design of drugs that selectively target components of this system
has led to major advances in therapy for cardiovascular diseases
such as hypertension and heart failure
 Discussed in Unit 4 Drugs Used In Disorders of the Cardiovascular System
Marc Imhotep Cray, M.D. 12
Volume expansion
feedback control
Raff RB, Rawls SM, Beyzarov EP. Netter's Illustrated Pharmacology, Updated Edition. Saunders, 2014
Marc Imhotep Cray, M.D. 13
Volume contraction
feedback control
Raff RB, Rawls SM, Beyzarov EP. Netter's Illustrated Pharmacology, Updated Edition. Saunders, 2014
Marc Imhotep Cray, M.D.
General Principles of Diuretic Action
14
 Definition: A diuretic is any substance that increases urine
and solute excretion
 This wide definition includes substances not commonly
thought of as diuretics, e.g. water
 To be therapeutically useful a diuretic should
 increase output of sodium as well as of water because
diuretics are normally required to remove edema fluid,
composed of water and solutes (of which sodium is most
important)
Marc Imhotep Cray, M.D.
GP of Diuretic Action (2)
15
Each day body produces 180 L of glomerular filtrate which is
modified in its passage down renal tubules to appear as 1.5 L of
urine
 Thus, if reabsorption of tubular fluid falls by 1%, urine output doubles
 Most clinically useful diuretics are organic anions
transported directly from blood into tubular fluid
 Following is a brief account of tubular function with particular
reference to sodium transport
 Intended to help to explain where and how diuretic drugs act
o it should be understood with reference to Figure following text
Marc Imhotep Cray, M.D.
GP of Diuretic Action (3)
16
Sites and modes of action
Proximal convoluted tubule (PCT)
 Some 65% of filtered sodium is actively transported from lumen of PCT by
sodium pump (Na+, K+-ATPase)
 Chloride is absorbed passively, accompanying sodium
 Bicarbonate is also absorbed through an action involving carbonic
anhydrase
 These solute shifts give rise to iso-osmotic reabsorption of water with
result that more than 70% of glomerular filtrate is returned to blood
from this section of nephron
 Epithelium of PCT is described as “leaky” because of its free permeability
to water and a number of solutes
Marc Imhotep Cray, M.D.
GP of Diuretic Action (4)
17
Sites and modes of action
Proximal convoluted tubule cont.
 Osmotic diuretics such as mannitol are non-resorbable
solutes which retain water in tubular fluid (Site 1 in Figure)
 Their effect is to increase water rather than sodium loss
reflected in their special use acutely to reduce intracranial
or intraocular pressure and not states associated with
sodium overload
Marc Imhotep Cray, M.D.
GP of Diuretic Action (5)
18
 Tubular fluid now passes into loop of Henle where 25% of
filtered sodium is reabsorbed
 There are two populations of nephron:
 those with short loops confined to cortex, and
 juxtamedullary nephrons whose long loops penetrate
deep into medulla  are concerned principally with water
conservation
o following discussion refers to these long loops
Sites and modes of action
Loop of Henle
Marc Imhotep Cray, M.D.
GP of Diuretic Action (6)
19
 Physiologic changes best understood by first considering ascending limb
 In TAL (Site 2, slide 25) sodium and chloride ions are transported from
tubular fluid into interstitial fluid by the three-ion co-transporter
system (i.e. Na+/K+/2Cl- called NKCC2) driven by sodium pump
o dependent on potassium returning to lumen through rectifying
outer medullary potassium (ROMK) channel otherwise K+ would
be rate limiting
 As tubule epithelium is “tight” here, i.e. impermeable to water tubular
fluid becomes dilute interstitium becomes hypertonic and
 fluid in adjacent descending limb, which is permeable to water
becomes more concentrated as it approaches tip of loop
o b/c hypertonic interstitial fluid sucks water out of this limb of tubule
Sites and modes of action
Loop of Henle cont.
Marc Imhotep Cray, M.D. 20
GP of Diuretic Action (7)
Sites and modes of action, Loop of Henle cont.
 High osmotic pressure in medullary interstitium is sustained by
descending and ascending vasa recta (long blood vessels of
capillary thickness that lie close to loops of Henle and act as
countercurrent exchangers) for incoming bld receives sodium
from outgoing bld
 Furosemide, bumetanide, torasemide and ethacrynic acid act
principally at TAL (Site 2, slide 25) by inhibiting the three-ion
transporter (NKCC2) thus preventing sodium ion reabsorption
and lowering osmotic gradient betw. cortex and medulla
results in formation of large volumes of dilute urine
 Hence, called “loop” diuretics
Marc Imhotep Cray, M.D. 21
GP of Diuretic Action (8)
Sites and modes of action
Distal convoluted tubule (DCT)
 Ascending limb of the loop then re-enters renal cortex where its
morphology changes into thin-walled DCT (Site 3, slide 25)
 Here uptake is still driven by sodium pump but sodium and chloride are
taken up through a different transporter Na-Cl cotransporter, called
NCC (formerly NCCT)
 Both ions are rapidly removed from interstitium b/c cortical blood flow
is high and there are no vasa recta present
 Epithelium is also tight at Site 3 and consequently urine becomes
more dilute
 Thiazides act at this region of cortical diluting segment by blocking NCC
transporter
Marc Imhotep Cray, M.D. 22
 In collecting duct (Site 4, slide 25), Na ions are exchanged for K and
H ions
 Na ions enter through epithelial Na channel (called ENaC), which
is stimulated by aldosterone
 The aldosterone (mineralocorticoid) receptor is inhibited by
competitive receptor antagonist spironolactone
whereas
 sodium channel is inhibited by amiloride and triamterene
 All three of these diuretics are potassium sparing b/c K+ is normally
secreted through K+ channel, ROMK (see Figure), down potential
gradient created by sodium reabsorption
GP of Diuretic Action (9)
Sites and modes of action
Cortical collecting duct (CCD)
Marc Imhotep Cray, M.D. 23
 All other diuretics, acting proximal to Site 4, cause potassium
Loss b/c they dump sodium into collecting duct
 Removal of this sodium through ENaC increases potential
gradient for potassium secretion through ROMK
 K+ sparing diuretics are weak diuretics b/c Site 4 is normally
responsible for “only” 2–3% of sodium reabsorption
 cause less sodium loss than thiazides or loop diuretics
 NB: Although ENaC does not have capacity to compensate for lg. Na
losses (e.g. loop diuretic usage) it is main site of physiologic control (via
aldosterone) over sodium loss
GP of Diuretic Action (10)
Sites and modes of action
Cortical collecting duct (CCD) cont.
Marc Imhotep Cray, M.D. 24
 Collecting ducts then travels back through medulla to reach
papilla in doing so it passes through a gradient of increasing
osmotic pressure which draws water out of tubular fluid
 This final conc. of urine is under influence of ADH =
increases water permeability by increasing expression of
specific water channels (or aquaporins)
o In ADH’s absence water remains in collecting duct
 Ethanol causes diuresis by inhibiting release of ADH from
posterior pituitary
GP of Diuretic Action (11)
Sites and modes of action
Cortical collecting duct (CCD) cont.
NB: Diuresis may also be achieved by extrarenal
mechanisms, by raising cardiac output and increasing
renal blood flow, e.g. with dobutamine and dopamine.
25Bennett PN, Brown MJ and Sharma P. Clinical Pharmacology 11th Ed. Edinburgh: Churchill Livingstone, 2012.
Sites of action of diuretic drugs
Inset show transporters and ion
channels targeted in tubular
cells at these sites
• ENaC, epithelial sodium
channel;
• NCCT, thiazide-sensitive
Na–Cl co-transporter;
• NKCC2, Na–K–2Cl co-
transporter;
• ROMK, rectifying outer
medullary potassium channel
Marc Imhotep Cray, M.D.
GP of Diuretic Action (12)
Classification
26
 Maximum efficacy in removing salt and water that any
diuretic achieves is dependent on its site of action, thus it is
appropriate to rank diuretics according to their natriuretic
capacity (as set out in slides that follow)
 Classes:
1. High efficacy
2. Moderate efficacy
3. Low efficacy
NB: Percentages refer to highest fractional excretion of filtered
sodium under carefully controlled conditions and should not be taken
to represent average fractional sodium loss during clinical use.
Marc Imhotep Cray, M.D.
GP of Diuretic Action (13)
Classification
27
1. High efficacy
Furosemide and other “loop” diuretics can cause up to 25% of
filtered sodium to be excreted
 Their action impairs powerful urine-concentrating mechanism of loop
of Henle and confers higher efficacy compared with drugs that act in
relatively hypotonic cortex
Progressive increase in dose is matched by increasing diuresis,
 i.e. they have a “high ceiling” of effect
 they are so effective that over-treatment can readily dehydrate patient
Loop diuretics remain effective at a glomerular filtration rate
(GFR) below 10 mL/min (nml 120 mL/min)
Marc Imhotep Cray, M.D.
GP of Diuretic Action (14)
Classification
28
2. Moderate efficacy
 The thiazide family, including chlorthalidone, chlorothiazide,
hydrochlorothiazide, metolazone and indapamide, cause 5-
10% of filtered sodium load to be excreted
 Increasing dose produces relatively little added diuresis
compared to loop diuretics
 i.e. they have a “low ceiling” of effect
 Cease to be effective once GFR has fallen below 20 mL/min
(except metolazone)
Marc Imhotep Cray, M.D.
GP of Diuretic Action (15)
Classification
29
3. Low efficacy
Triamterene, amiloride and spironolactone cause 2–3% of
filtered sodium to be excreted
 They are potassium sparing and combine with more
efficacious diuretics to prevent potassium loss, which other
diuretics cause
Osmotic diuretics, e.g. mannitol, also fall into this category
Marc Imhotep Cray, M.D.
GP of Diuretic Action (16)
Indications
30
 Edema states associated
 with sodium overload, e.g. cardiac, renal or hepatic disease, and
also
 without sodium overload, e.g. acute pulmonary edema following
myocardial infarction
NB: Edema may also be localized, e.g.
 angioedema over face and neck or around ankles with some
calcium channel blockers, or
 due to low plasma albumin, or immobility in elderly
 in none of these circumstances is a diuretic indicated
 Hypertension, by reducing intravascular volume and other mechanisms
too, e.g. reduction of sensitivity to noradrenergic vasoconstriction
Marc Imhotep Cray, M.D. 31
 Hypercalcemia Furosemide reduces calcium reabsorption in ascending limb
of loop of Henle action may be utilised in emergency reduction of raised
plasma calcium levels, in addition to rehydration and other measures
 Idiopathic hypercalciuria, a common cause of renal stone disease, may be
reduced by thiazide diuretics
 Syndrome of inappropriate secretion of antidiuretic hormone secretion
(SIADH) may be treated with furosemide if there is a dangerous degree of
volume overload
 Nephrogenic diabetes insipidus, paradoxically, may respond to diuretics
which, by contracting vascular volume, increase salt and water reabsorption
in PCT thus reduce urine volume
GP of Diuretic Action (17)
Indications cont.
Marc Imhotep Cray, M.D.
Individual Agents/Classes
32
High-efficacy (loop) diuretics
Furosemide…
Moderate-efficacy diuretics
Thiazides…
Low-efficacy (K+ sparing) diuretics
Spironolactone…
Osmotic diuretics
Mannitol…
Carbonic Anhydrase Inhibitors
Acetazolamide…
Marc Imhotep Cray, M.D.
High-efficacy (loop) diuretics
Furosemide (Prototype)
Furosemide acts on thick portion of ascending limb of the loop
of Henle (Site 2 in slide 25) to produce effects described above
 b/c more sodium is delivered to DCT & CD (Site 4 in slide 25), exchange
with potassium leads to urinary potassium loss and hypokalemia
 Magnesium and calcium loss are increased by furosemide to same
extent as sodium effect on calcium is utilized in emergency
management of hypercalcemia
Pharmacokinetics
 Absorption from GIT is subject to considerable intra- and inter-
individual variation and it is highly bound to plasma proteins
 t½ is 2 hrs rises to over 10 h in renal failure
33
Marc Imhotep Cray, M.D.
High-efficacy (loop) diuretics
Furosemide cont.
Uses
 very successful for the relief of edema
 Urine production rises progressively with increasing dose
 Taken orally it acts within an hour and diuresis lasts up to 6 h
Caution
 Enormous urine volumes can result and over-treatment may lead to
hypovolemia and circulatory collapse
 Given intravenously it acts within 30 min and can relieve acute pulmonary
Edema partly by a venodilator action which precedes diuresis
 Important feature  retains efficacy even at a low GFR (10 mL/min or less)
34
Marc Imhotep Cray, M.D.
High-efficacy (loop) diuretics
Furosemide cont.
Adverse effects
uncommon, apart from excess of therapeutic effect (electrolyte
disturbance and hypotension due to low plasma volume) and
 Nausea
 Pancreatitis and,
 rarely, deafness, which is usually transient and associated
with rapid IV injection in renal failure
Non-steroidal anti-inflammatory drugs (NSAIDs), notably
indomethacin, reduce furosemide-induced diuresis by inhibiting
formation of vasodilator prostaglandins in kidney
35
Marc Imhotep Cray, M.D.
High-efficacy (loop) diuretics cont.
Bumetanide, piretanide and ethacrynic acid are similar to
furosemide
Bumetanide may be preferred over furosemide
in heart failure b/c of more predictable oral absorption
Ethacrynic acid is less widely used as it is more prone to cause
adverse effects, especially nausea and deafness (ototoxicity)
 Not sulfa containing as are other loop diuretics, thus useful option in
sulfa-allergic pts.
Torasemide is an effective antihypertensive agent at lower
(non-natriuretic) doses (2.5–5 mg/day) than those used for
edema (5–40 mg/day)
36
Marc Imhotep Cray, M.D.
NB Sidebar: Sulfa drugs and sulfa allergies
 Sulfa containing drugs:
Sulfonamides antibiotics, Sulfasalazine, Probenecid, Furosemide,
Acetazolamide, Celecoxib, Thiazides, Sulfonylureas
(Scary Sulfa Pharm FACTS)
 Clinical manifestations of sulfa allergies= Pts. w sulfa allergies
may develop:
 Fever
 Urinary tract infection
 Stevens-Johnson syndrome (SJS)
 Hemolytic anemia, thrombocytopenia agranulocytosis, and
 Urticaria (hives)
 Symptoms range from mild to life threatening
Marc Imhotep Cray, M.D.
Moderate-efficacy diuretics
Thiazides
Thiazides depress salt reabsorption in DCT (Site 3 in slide 25),
i.e. upstream of region of sodium–potassium exchange at CD
(Site 4 in slide 25)
 Hence , have important effect of raising potassium excretion
 Thiazides lower blood pressure, initially due to a reduction in
intravascular volume but chronically by a reduction in
peripheral vascular resistance
 accompanied by diminished responsiveness of vascular
smooth muscle to Epi/NE
 also have a direct action on vascular smooth muscle
membranes
37
Marc Imhotep Cray, M.D.
Moderate-efficacy diuretics
Thiazides cont.
Uses
 given for mild cardiac failure and mild hypertension, or for more severe
degrees of HTN, in combination with other drugs
Pharmacokinetics
 Thiazides are well absorbed orally and most begin to act within an hour
 Differences among numerous derivatives lie in duration of action
 Relatively water-soluble agents, e.g. chlorothiazide,
hydrochlorothiazide (HCTZ), are most rapidly eliminated, peak effect
within 4–6 h and passing off by 10–12 h
o excreted unchanged in urine and active secretion by PCT
contributes to high renal clearance and t½ of less than 4 h
38
Marc Imhotep Cray, M.D.
Moderate-efficacy diuretics
Thiazides cont.
Pharmacokinetics
 Relatively lipid-soluble members, e.g. polythiazide,
hydroflumethiazide, distribute more widely into body tissues
and act for >24 h
o can be problematic if used for diuresis, but no evidence this property
makes them more effective at controlling hypertension
With exception of metolazone, thiazides are not effective when
renal function is impaired (GFR <20 mL/min), b/c they are not
filtered in sufficient conc. to inhibit NCC (Na-Cl cotransporter)
39
Marc Imhotep Cray, M.D.
Moderate-efficacy diuretics
Thiazides cont.
Adverse effects
(Adverse effects in general discussed in a section to follow)
 Rashes (sometimes photosensitive)
 thrombocytopenia and
 agranulocytosis occur
 Thiazide-type drugs can increase total plasma cholesterol concentration
 But in long-term use this is less than 5%, even at high doses
 Questions about appropriateness of thiazides for mild hypertension, of
which ischemic heart disease is a common complication, are laid to rest
by their proven success in randomized outcome comparisons
40
Marc Imhotep Cray, M.D.
Moderate-efficacy diuretics
Diuretics related to thiazides
Several compounds, not strictly thiazides, share structural similarities and
act at same site on nephron
Overall, these agents have a longer duration of action, are used for
edema and hypertension, and their profile of adverse effects similar to
thiazides
 Chlorthalidone acts for 48–72 h after a single oral dose
 Indapamide is structurally related to chlortalidone but lowers blood
pressure at subdiuretic doses perhaps by altering calcium flux in
vascular smooth muscle
 Metolazone is effective when renal function is impaired
o It potentiates diuresis produced by furosemide and combination can
be effective in resistant edema although risk of hypokalemia is
very high 41
Marc Imhotep Cray, M.D.
Low-efficacy diuretics
 Spironolactone (Aldactone) is structurally similar to aldosterone and
competitively inhibits its action in distal tubule (exchange of potassium
for sodium, Site 4 of slide 25)
 Excessive secretion of aldosterone contributes to fluid retention in
 hepatic cirrhosis
 nephrotic syndrome
 congestive heart failure and
 primary hypersecretion (Conn’s syndrome)
 Spironolactone is also useful in treatment of resistant hypertension
 increased aldosterone sensitivity is increasingly recognized as a
contributory factor
42
Marc Imhotep Cray, M.D.
Low-efficacy diuretics cont.
 Spironolactone has a short t½ (1.6 h), being extensively metabolized, and its
prolonged diuretic effect is due to most significant active metabolite,
canrenone (t½ 17 h)
 relatively ineffective when used alone
 more efficient when combined w a drug that reduces sodium
 given orally in one or more doses totaling 100–200 mg/day
 Maximum diuresis may not occur for up to 4 days
 Spironolactone (and amiloride and triamterene) usefully reduces K+ loss
caused by loop diuretics
Warnings:
 combination with another K+ sparing diuretic must be avoided as
hyperkalemia will result
 Dangerous K+ retention is particularly likely if spironolactone is given to pts.
with impaired renal function 43
Marc Imhotep Cray, M.D.
Low-efficacy diuretics cont.
Adverse effects
 Estrogenic effects are major limitation to its long-term use
 Randomized Aldactone Evaluation Study (RALES) even 25 mg/day caused breast
tenderness or enlargement in 10% of men (N Engl J Med. 1999 Sep 2;341(10):709-17)
 Women may also report breast discomfort or menstrual irregularities, including
amenorrhea
 Minor gastrointestinal upset and increased risk of gastroduodenal ulcer
and bleeding
 Usually reversible on stopping drug
 Spironolactone is reported to be carcinogenic in rodents, but clinical
experience suggest tit is safe in humans
 Nevertheless, UK license for its use in essential hypertension was withdrawn (i.e.
possible use long term in a pt. group including relatively young), but is retained for
other indications 44
Marc Imhotep Cray, M.D.
Low-efficacy diuretics cont.
 Eplerenone is a spironolactone analog licensed for use
in heart failure
 free of estrogenic effects; b/c of its lower affinity for
estrogen receptor
 It is useful in pts who need an aldosterone-receptor blocking
agent, but are intolerant of endocrine effects of spironolactone
45
Marc Imhotep Cray, M.D.
Low-efficacy diuretics cont.
Amiloride blocks ENaC sodium channels in distal tubule
 Action complements thiazides with which it is frequently
combined to increase sodium loss and limit potassium loss
 Example, coamilozide (amiloride 2.5–5 mg plus
hydrochlorothiazide 25–50 mg) is used for hypertension or
edema
maximum effect of amiloride occurs about 6 h after an oral
dose, with a duration of action greater than 24 h (t½ 21 h)
oral dose is 5–20 mg daily
46
Marc Imhotep Cray, M.D.
Low-efficacy diuretics cont.
 Triamterene (Dytac) is a potassium-sparing diuretic with
an action and use similar to amiloride (blocks ENaC sodium
channels in DCT)
 Diuretic effect extends over 10 h
Adverse effects
 Gastrointestinal upsets occur
Drug-drug interaction
 Reversible, non-oliguric renal failure may occur when
triamterene is used with indomethacin (and other NSAIDs)
 may also give urine a blue coloration
47
Marc Imhotep Cray, M.D.
Osmotic diuretics
 Osmotic diuretics are small molecular weight substances that are
filtered by the glomerulus but not reabsorbed by renal tubule
and thus increase osmolarity of tubular fluid
 Thus they prevent reabsorption of water (and also, by more
complex mechanisms, of sodium) principally in PCT and also loop
of Henle
 Result is urine volume increases according to load of osmotic
diuretic
48
Marc Imhotep Cray, M.D.
Osmotic diuretics cont.
 Mannitol, a polyhydric alcohol (mol. wt. 452), is used most commonly
given intravenously
 In addition to effect on kidney, mannitol encourages movement of water
from inside cells to extracellular fluid ECF (including circulatory volume)
thus transiently expanded before diuresis occurs
Uses:
Rapid reduction of intracranial or intraocular pressure, and to maintain urine
flow to prevent renal tubular necrosis
Contraindications:
 b/c mannitol increases circulatory volume, it is contraindicated in
congestive cardiac failure (CHF) and pulmonary edema (PE)
49
Marc Imhotep Cray, M.D.
Carbonic Anhydrase Inhibitors
 The enzyme carbonic anhydrase facilitates reaction betw. CO2 and H2O to
form carbonic acid (H2CO3), which then breaks down to hydrogen (H+) and
bicarbonate (HCO3-) ions
 This process is fundamental to production of either acid or alkaline
secretions
 high concentrations of CA are present in gastric mucosa, pancreas, eye and kidney
 MOA b/c number of H+ ions available to exchange with Na+ in PCT is
reduced, sodium loss and diuresis occur
 But HCO3- reabsorption from tubule is also reduced, and its loss in urine
leads within days to metabolic acidosis attenuates diuretic response
to carbonic anhydrase inhibition
o Consequently, inhibitors of CA are obsolete as diuretics
• Still have specific uses
 Acetazolamide is most widely used CAI 50
Marc Imhotep Cray, M.D.
Carbonic Anhydrase Inhibitors cont.
Reduction of intraocular pressure
 action is not due to diuresis  rather, formation of aqueous humor is
an active process requiring a supply of bicarbonate ions which depends
on carbonic anhydrase
 Inhibition of CA reduces formation of aqueous humor and lowers IOP
o this is a local action and is not affected by development of acid–base
changes elsewhere in body, i.e. tolerance does not develop
 In pts. w acute glaucoma, acetazolamide taken either PO or IV
 Acetazolamide is not recommended for long-term use b/c of risk of
hypokalemia and acidosis but brinzolamide or dorzolamide are effective
as eye drops, well tolerated, and thus suitable for chronic use in glaucoma
51
Marc Imhotep Cray, M.D.
Carbonic Anhydrase Inhibitors cont.
Acetazolamide for High-altitude (mountain) sickness
 High-altitude (mountain) sickness may affect unacclimatized people at
altitudes over 3000 meters, especially after rapid ascent
 symptoms range from
 nausea
 lassitude and headache to
 pulmonary and cerebral edema
 Initiating cause is hypoxia:
 at high altitude, normal hyperventilatory response to falling oxygen
tension is inhibited b/c alkalosis is also induced
 Acetazolamide induces metabolic acidosis increases respiratory drive,
notably at night when apnetic attacks may occur, and thus helps to maintain
arterial oxygen tension 52
Marc Imhotep Cray, M.D.
CAIs cont., acetazolamide for high-altitude
Dosage
 Usual dose is 125–250 mg twice daily, given orally on day
before ascent and continued for 2 days after reaching
intended altitude
 250 mg twice daily is used to treat established high-altitude
sickness, combined with a return to a lower altitude
(Note: this is an unlicensed indication in UK)
 As an alternative or in addition to acetazolamide
dexamethasone may be used:
 2 mg q6 hrs. for prevention, and
 4 mg q6 hrs. for treatment 53
Marc Imhotep Cray, M.D.
CAIs cont., acetazolamide
Acetazolamide has two other uses
1. In periodic paralysis, where sudden falls in plasma K+ conc.
occur due to its exchange with Na+ in cells
 rise in plasma H+ caused by acetazolamide provides an
alternative cation to K+ for exchange with Na+
2. Acetazolamide may be used occasionally as a second-line
drug for tonic–clonic and partial epileptic seizures
54
Marc Imhotep Cray, M.D.
CAIs cont., acetazolamide
Adverse effects
 High doses of acetazolamide may cause
 drowsiness and fever
 rashes (it is a sulfonamide-type drug) and
 paranesthesia may occur (from the acidosis)
 blood disorders have been reported
 Renal calculi may develop, b/c urine calcium is in less
soluble form, owing to low citrate content of urine a
consequence of metabolic acidosis
 Dichlorphenamide is a similar, but a more potent, inhibitor
of carbonic anhydrase
55
Marc Imhotep Cray, M.D.
Adverse effects of diuretics
and
Drug-Drug Interactions
57
Marc Imhotep Cray, M.D.
Potassium depletion
57
Diuretics that act at Sites 1, 2 and 3 of slide 25 cause more
sodium to reach sodium–potassium exchange site in distal tubule
(Site 4) and so increase potassium excretion
This subject warrants discussion b/c hypokalemia may cause
cardiac arrhythmia in patients at risk (e.g. receiving digoxin)
Safe lower limit for plasma potassium conc. is 3.5 mEq/L
Whether or not diuretic therapy causes significant lowering of
serum potassium levels depends both on drug and on
circumstances in which it is used following slides explain more
Marc Imhotep Cray, M.D.
Potassium depletion
58
 The loop diuretics produce a smaller fall in serum K+ conc. than
do thiazides, for equivalent diuretic effect, but have a greater
capacity for diuresis, i.e. higher efficacy especially in large
dose so are associated with greater decline in potassium
levels
 If diuresis is brisk and continuous, clinically important
potassium depletion is likely to occur
 NB: Hypokalemia predisposes pts Tx with cardiac glycosides
(digoxin) to toxicity (permissive for digoxin binding at K+ binding
site on Na+/K+ ATPase)
Marc Imhotep Cray, M.D.
Potassium depletion cont.
59
 Low dietary intake of potassium predisposes to hypokalemia
risk is particularly notable in elderly, many of whom ingest less
than 50 mEq per day (dietary normal is 80 mEq).
 Hypokalemia may be aggravated by other drugs, e.g. β2-
agonists, theophylline, corticosteroids, amphotericin
 Hypokalemia during diuretic therapy is also more likely in
hyperaldosteronism
 whether primary or more commonly secondary to severe
liver disease, congestive heart failure or nephrotic
syndrome
Marc Imhotep Cray, M.D.
Potassium depletion cont.
 Potassium loss occurs with diarrhea, vomiting and small bowel
fistula and may be aggravated by diuretic therapy
 When a thiazide diuretic is used for hypertension no case for
routine prescription of a potassium supplement if no
predisposing factors are present
60
Marc Imhotep Cray, M.D.
Potassium depletion cont.
Potassium depletion can be minimized or corrected by:
 Maintaining a good dietary potassium intake (fruits, fruit juices,
vegetables)
 Combining a potassium-depleting with a potassium sparing agent
 Intermittent use of potassium-losing drugs, i.e. drug holidays
 Potassium supplements: KCl preferred b/c chloride is principal anion
excreted along with sodium when high-efficacy diuretics are used
Potassium-sparing diuretics defend plasma potassium more
effectively than potassium supplements
NB: All forms of K are irritant to GIT, and in esophagus may cause ulceration.
Elderly, in particular, should be warned never to take such tablets dry but
always with a large cupful of liquid and sitting upright or standing. 61
Marc Imhotep Cray, M.D.
Hyperkalemia
Hyperkalemia may occur, esp. if a K+ sparing diuretic is given to
a patient with impaired renal function
 ACE inhibitors and ARBs can cause a increase in plasma K+ levels
 They may cause dangerous hyperkalemia if combined with
KCl supplements or other potassium sparing drugs, in
presence of impaired renal function
o However, w suitable monitoring combination can be used safely,
as was well illustrated by RALES trial
 Cyclosporine, tacrolimus, indomethacin and possibly other
NSAIDs may cause hyperkalemia w potassium-sparing diuretics
62
Marc Imhotep Cray, M.D.
Treatment of hyperkalemia
Tx of ↑K+ depends on severity and following measures are
appropriate:
 Any potassium-sparing diuretic should be discontinued
 A cation-exchange resin, e.g. Polystyrene sulfonate resin can
be used orally (more effective than rectally), to remove body
potassium by gut
 K+ may be moved rapidly from plasma into cells by giving:
o sodium bicarbonate, 50 mL 8.4% solution through a central line, and
repeated in a few minutes if characteristic ECG changes persist
o glucose, 50 mL 50% solution, plus 10 units regular insulin by i.v.
infusion
o nebulized β2-agonist, salbutamol 5–10 mg, is effective in stimulating
pumping of potassium into skeletal muscle
Marc Imhotep Cray, M.D.
Tx of hyperkalemia cont.
 In presence of ECG changes, calcium gluconate, 10 mL of 10%
solution, given i.v. and repeated if necessary in a few minutes
o it has no effect on serum potassium but opposes
myocardial effect of a raised serum potassium level
o Caution
• Calcium may potentiate digoxin and should be used cautiously, if
at all, in a patient taking this drug
• NB: Sodium bicarbonate and calcium salt must not be mixed in a
syringe or reservoir b/c calcium precipitates
 Dialysis may be needed in refractory cases & is highly effective
Marc Imhotep Cray, M.D.
Hypovolemia
 Hypovolemia can result from over-treatment
 Acute loss of excessive fluid leads to postural hypotension
and dizziness
 A more insidious state of chronic hypovolemia can develop,
especially in elderly
 After initial benefit, pt. becomes sleepy and lethargic
 Blood urea concentration (BUN) rises and Na+ conc. may be
low
o Renal failure may result
63
Marc Imhotep Cray, M.D.
Urinary retention
Urinary retention
 Sudden vigorous diuresis can cause acute retention of urine in
presence of bladder neck obstruction
 e.g. due to prostatic enlargement
64
Marc Imhotep Cray, M.D.
Hyponatremia
 Hyponatremia may result if Na+ loss occurs in pts who drink a large quantity
of water when taking a diuretic
 Other mechanisms are involved, including enhancement of ADH release
 Such pts. have reduced total body Na+ and ECF vol. and are edema free
 Discontinuing diuretic and restricting water intake are effective
 The condition should be distinguished from hyponatremia with edema,
which develops in patients with CHF, cirrhosis or nephrotic syndrome
 Here salt and water intake should be restricted b/c ECF volume is expanded
 Combination of a potassium-sparing diuretic and ACE inhibitor can also
cause severe hyponatremia  more commonly than life-threatening
hyperkalemia
65
Marc Imhotep Cray, M.D.
Urate retention
Urate retention with hyperuricemia and, sometimes, clinical gout occurs
with thiazides and loop diuretics
 Effect is unimportant or negligible with low-efficacy diuretics, e.g.
amiloride and spironolactone
 Two mechanisms responsible
 First, diuretics cause volume depletion, reduction in glomerular
filtration and increased absorption of almost all solutes in proximal
tubule, including urate
 Second, diuretics and uric acid are organic acids and compete for
transport mechanism that pumps such substances from blood into
tubular fluid
 Diuretic-induced hyperuricemia can be prevented by allopurinol or
probenecid (which also antagonizes diuretic efficacy by reducing their
transport into urine)
66
Marc Imhotep Cray, M.D.
Magnesium deficiency
Magnesium deficiency: Loop and thiazide diuretics cause
significant urinary loss of magnesium
 potassium-sparing diuretics cause magnesium retention
Magnesium deficiency brought about by diuretics is rarely
severe enough to induce classic picture of neuromuscular
irritability and tetany  but cardiac arrhythmias, mainly of
ventricular origin, do occur
 respond to repletion of magnesium (2 g of Mg2+ is given as
4 mL 50% magnesium sulfate infused i.v. over 10–15 min
followed by up to 70 mmol infused over the next 24 h)
67
Marc Imhotep Cray, M.D.
Carbohydrate intolerance
Carbohydrate intolerance is caused by those diuretics that
produce prolonged hypokalemia, i.e. loop and thiazide type
Mechanism
May affect depolarization and entry of calcium into islet cells
which is necessary to stimulate formation and release of
insulin so glucose intolerance is probably due to secondary
insulin deficiency
 Insulin requirements thus increase in established diabetics and
disease may become manifest in latent diabetics
 effect is generally reversible over several months
68
Marc Imhotep Cray, M.D.
Calcium homeostasis
Renal calcium loss is increased by loop diuretics
 In short term this is not a serious disadvantage and furosemide may be
used in management of hypercalcemia after rehydration achieved
 In long term hypocalcaemia may be harmful, especially in elderly
patients, who tend in any case to be in negative calcium balance
Thiazides, by contrast, decrease renal excretion of calcium
 this property may influence choice of diuretic in a potentially calcium-
deficient or osteoporotic individual as thiazide use is associated with
a reduced risk of hip fracture in elderly
 Hypocalciuric effect of thiazides has also been used effectively in
patients with idiopathic hypercalciuria commonest metabolic cause
of renal stones
69
Marc Imhotep Cray, M.D.
Drug-Drug Interactions
 Loop diuretics (especially as intravenous boluses) potentiate ototoxicity of
aminoglycosides and nephrotoxicity of some cephalosporins
 NSAIDs tend to cause sodium retention, which counteracts the effect of
diuretics mechanism may involve inhibition of renal prostaglandin
formation
 Diuretic treatment of a patient taking lithium can precipitate toxicity from
this drug (increased sodium loss is accompanied by reduced lithium
excretion)
 Other drugs that may induce hyperkaliemia, hypokalemia, hyponatremia or
glucose intolerance with diuretics are described above
70
Marc Imhotep Cray, M.D.
Alteration Of Urine pH
Alteration of urine pH by drugs is sometimes desirable  most
common reason is in treatment of poisoning
(a fuller account is given in poisoning and overdose)
A summary of main indications follows:
 Alkalinization of urine:
 increases elimination of salicylate, phenobarbital and
chlorophenoxy herbicides
 treats crystal nephropathy by increasing drug solubility, e.g.
of methotrexate, sulfonamides and triamterene
 reduces irritation of an inflamed urinary tract
 discourages growth of certain organisms, e.g. Escherichia coli
71
Marc Imhotep Cray, M.D.
Alteration Of Urine pH cont.
 Urine can be made alkaline by sodium bicarbonate i.v., or by potassium citrate
by mouth
Caution: Sodium overload may exacerbate cardiac failure, and sodium or
potassium excess are dangerous when renal function is impaired
Acidification of urine:
 used as a test for renal tubular acidosis
 increases elimination of amphetamine, MDMA or “Ecstasy”, quinine and
phencyclidine (very rarely needed)
 Oral NH4Cl, taken w food to avoid vomiting, acidifies urine
o It should not be given to pts with impaired renal or hepatic
function
 Other means include arginine hydrochloride, ascorbic acid and
calcium chloride by mouth 72
Marc Imhotep Cray, M.D. 76
Drugs and the Kidney
and
Prescribing in Renal Disease
Marc Imhotep Cray, M.D.
Drugs and the Kidney
Drug-induced renal disease
Drugs and other chemicals damage kidney by 3 major mechanisms:
1. Direct biochemical effect Substances that cause such toxicity include:
 heavy metals, e.g. mercury, gold, iron, lead
 antimicrobials, e.g. aminoglycosides, amphotericin, cephalosporins
 iodinated radiological contrast media, e.g. agents for visualizing GIT
 analgesics, e.g. NSAID combinations
 solvents, e.g. carbon tetrachloride, ethylene glycol
2. Indirect biochemical effect:
 cytotoxic drugs and uricosurics may cause urate to be precipitated in
tubule
 calciferol may cause renal calcification by inducing hypercalcemia
 diuretic and laxative abuse can cause tubular damage secondary to K+
and Na+ depletion
 anticoagulants may cause hemorrhage into kidney
Marc Imhotep Cray, M.D.
Drugs and the Kidney (2)
Drug-induced renal disease cont.
3. Immunological effect
 A wide range of drugs produces a wide range of injuries:
 including phenytoin, gold, penicillins, hydralazine, isoniazid, rifampin,
penicillamine, probenecid, sulfonamides
 drugs may cause damage by more than one of above mechanisms, e.g. gold
 Sites and pathological types of injury are:
o Glomerular damage, eg. Penicillamine= damage from circulating
immune complexes
o Tubule damage, eg. acids, e.g. salicylate (aspirin), cephalosporins &
bases, e.g. aminoglycosides, Heavy metals and contrast media
o Tubule obstruction, eg. Methotrexate is relatively insoluble at low
urine pH and can precipitate in distal nephron
Drugs and the Kidney (3)
 Other drug-induced lesions of kidney include:
 Vasculitis, caused by allopurinol, isoniazid, sulfonamides
 Allergic interstitial nephritis, caused by penicillins (especially), thiazides, allopurinol,
phenytoin, sulfonamides
 Drug-induced lupus erythematosus, caused by hydralazine, procainamide,
sulfasalazine
 Drugs may thus induce any of common clinical syndromes of renal injury, namely:
 Acute renal failure, e.g. aminoglycosides, cisplatin
 Nephrotic syndrome, e.g. penicillamine, gold, captopril (only at higher doses than
now recommended)
 Chronic renal failure, e.g. NSAIDs.
 Functional impairment, i.e. reduced ability to dilute and concentrate urine (lithium),
potassium loss in urine (loop diuretics), acid–base imbalance (acetazolamide)
Marc Imhotep Cray, M.D.
Prescribing In Renal Disease
 Drugs may:
 exacerbate renal disease (see previous section)
 be ineffective, e.g. thiazide diuretics in moderate or severe renal failure;
uricosurics
 be potentiated by accumulation due to failure of renal excretion
 First option is to seek an alternative drug that does not depend on renal
elimination
 NB: Problems of safety arise for pts. w impaired renal function who must
be treated w a drug that is potentially toxic and that is wholly or largely
eliminated by kidney
 A knowledge of, or at least access to, sources of pharmacokinetic data
is essential for safe therapy for such patients,
o e.g. manufacturers’ data, formularies and specialist journals
Marc Imhotep Cray, M.D.
Prescribing In Renal Disease (2)
Drug t½ (h) in normal and severely impaired renal function
Glomerular filtration rate <5 mL/min (normal value is 120 mL/min).
These values illustrate major effect of impaired renal function on
elimination of certain drugs. Depending on circumstances, alternative
drugs must be found or special care exercised when prescribing drugs
that depend significantly on kidney for elimination.
Modifiedafter:BennettPN,BrownMJandSharmaP.Clinical
Pharmacology11thEd.Edinburgh:ChurchillLivingstone,2012.
Marc Imhotep Cray, M.D.
Prescribing In Renal Disease (3)
t½ of other drugs, where activity is terminated by metabolism,
is unaltered by renal impairment
  but many such drugs produce pharmacologically active metabolites
that are more water soluble than parent drug rely on kidney for their
elimination, and accumulate in renal failure, e.g.
o acebutolol,
o diazepam,
o warfarin,
o meperidine
Majority of drugs fall into an intermediate class =
 partly metabolized and
 partly eliminated unchanged by kidney
Marc Imhotep Cray, M.D.
Prescribing In Renal Disease (4)
 Administering correct dose to a patient with renal disease
must take into account both
1. extent drug normally relies on renal elimination and
2. degree of renal impairment
 Best guide to renal impairment is creatinine clearance and not
serum creatinine level itself as serum Cr can be notoriously
misleading in elderly and at extremes of body mass
 CrCl can be predicted from serum creatinine concentration, sex, age
and weight using Cockcroft–Gault formula
o A number of free online calculators are available, e.g. http://www.medical-
calculator.nl/calculator/GFR/
To learn more see: Tutorial_3 Renal Drug Elimination and Dosing in Renal Impairment
Marc Imhotep Cray, M.D.
Prescribing In Renal Disease (5)
Dose adjustment for patients with renal impairment
 Adjustment of initial dose (or where necessary priming or loading dose) is
generally unnecessary, as volume into which drug has to distribute should be
same in uremic and healthy subject
 There are exceptions to this rule of thumb for example, volume of distribution of
digoxin is contracted in urinemic patients due to altered tissue binding of drug
 Adjustment of maintenance dose involves either reducing each dose given
or lengthening time between doses
 Special caution is needed when patient is hypoproteinemia and drug is
usually extensively plasma protein bound, or in advanced renal disease
when accumulated metabolic products may compete for protein binding
sites
 Careful observation is required in early stages of dosing until response to
drug can be gauged
Prescribing In Renal Disease (5)
General rules
1. Drugs that are completely or largely excreted by kidney, or drugs that produce active,
renally eliminated metabolites:
 give a normal or, if there is special cause for caution (as discussed above), a slightly
reduced initial dose, and lower maintenance dose or lengthen dose interval in
proportion to reduction in creatinine clearance
2. Drugs that are completely or largely metabolized to inactive products:
 Give normal doses
 When note of special caution (as above) applies, a modest reduction of initial dose
and maintenance dose rate are justified while drug effects are assessed
3. Drugs that are partly eliminated by kidney and partly metabolized:
 give a normal initial dose and modify maintenance dose or dose interval in light of what
is known about patient’s renal function and drug its dependence on renal elimination
and its inherent toxicity
Marc Imhotep Cray, M.D.
Prescribing In Renal Disease (6)
Recall that time to reach steady-state blood concentration is
dependent only on drug t½, and a drug reaches 97% of its
ultimate steady-state concentration in 4-5 x t½
 Thus, if t½ is prolonged by renal impairment, so also will be the time to
reach steady state.
Schemes for modifying drug dosage for patients with renal
disease diminish but do not remove their increased risk of
adverse effects
 patients should be observed carefully throughout a course of drug TX
 Where service is available, dosing should be monitored by drug plasma
concentration measurements
Marc Imhotep Cray, M.D.
ADH Antagonists
87
 Demeclocycline
 Lithium
 Lixivaptan
 Satavaptan
 Conivaptan
 Tolvaptan
Marc Imhotep Cray, M.D.
Antidiuretic Hormone Antagonists
88
 A variety of medical conditions, including
 Congestive heart failure (CHF) and
 Syndrome of inappropriate ADH secretion (SIADH) cause
water retention as a result of excessive ADH secretion
o Inability to form dilute urine in fully hydrated condition is
characteristic of SIADH
• Antagonists of ADH are needed to treat this condition
 Patients with CHF who are on diuretics frequently develop
hyponatremia secondary to excessive ADH secretion
Dangerous hyponatremia can result
Marc Imhotep Cray, M.D.
Antidiuretic Hormone Antagonists (2)
89
Until recently, two nonselective agents
 lithium and
 demeclocycline (a tetracycline antimicrobial drug),
were used for their well-known interference with ADH activity
 Mechanism for this interference has not been completely
determined for either of these agents
 Demeclocycline is used more often than lithium because
of many adverse effects of lithium administration
 Demeclocycline is now being rapidly replaced by several
specific ADH receptor antagonists (vaptans), which have
yielded good clinical results
Marc Imhotep Cray, M.D.
Antidiuretic Hormone Antagonists (3)
90
 There are 3 known vasopressin receptors, V1a , V1b , and V2
 V1 receptors are expressed in vasculature and CNS
 V2 receptors are expressed specifically in kidney
 Conivaptan (currently available only for intravenous use)
exhibits activity against both V1a and V2 receptors
 Oral agents tolvaptan, lixivaptan, and satavaptan are
selectively active against V2 receptor
 Tolvaptan, is very effective in treatment of hyponatremia,
SIADH and as an adjunct to standard diuretic therapy in
patients with CHF
Vaptans
Marc Imhotep Cray, M.D.
Antidiuretic Hormone Antagonists (4)
91
Pharmacokinetics
 Half-life of conivaptan and demeclocycline is 5–10 hours,
while that of tolvaptan is 12–24 hours
Pharmacodynamics
 Antidiuretic hormone antagonists inhibit effects of ADH in
collecting tubule
 Conivaptan and tolvaptan are direct ADH receptor
antagonists
 both lithium and demeclocycline reduce ADH-induced
cAMP by mechanisms that are not completely yet clarified
Marc Imhotep Cray, M.D.
Antidiuretic Hormone Antagonists (5)
92
Clinical Indications & Dosage
A. Syndrome of Inappropriate ADH Secretion
 For SIADH, water restriction is often treatment of choice
 ADH antagonists are used to manage SIADH when water restriction has
failed to correct abnormality
 Generally occurs in outpatient setting, where water restriction cannot
be enforced, but Can occur in hospital when large quantities of
intravenous fluid are needed for other purposes
o Demeclocycline (600–1200 mg/d) or tolvaptan (15–60 mg/d) can
be used for SIADH
• Appropriate plasma levels of demeclocycline (2 mcg/mL)
should be maintained by monitoring
• Tolvaptan levels are not routinely monitored
Marc Imhotep Cray, M.D.
Antidiuretic Hormone Antagonists (6)
93
Clinical Indications cont.
B. Other Causes of Elevated Antidiuretic Hormone
 Antidiuretic hormone is also elevated in response to diminished effective
circulating blood volume, as often occurs in heart failure
 When Tx by volume replacement is not desirable, hyponatremia may
result
 As for SIADH, water restriction is often treatment of choice
o In patients with heart failure, this approach is often unsuccessful in
view of increased thirst and large number of oral medications being
used
 For patients with heart failure, intravenous conivaptan may be
particularly useful b/c it has been found that blockade of V1a receptors
leads to decreased peripheral vascular resistance and increased cardiac
output
Marc Imhotep Cray, M.D.
Antidiuretic Hormone Antagonists (7)
94
Toxicity
A. Nephrogenic Diabetes Insipidus
If serum Na + is not monitored closely, any ADH antagonist can
cause severe hypernatremia and nephrogenic diabetes insipidus
 If lithium is being used for a psychiatric disorder, nephrogenic
diabetes insipidus can be treated with a thiazide diuretic or
amiloride
B. Renal Failure
Both lithium and demeclocycline have been reported to cause
acute renal failure
 Long-term lithium therapy may cause chronic interstitial nephritis
Marc Imhotep Cray, M.D.
Antidiuretic Hormone Antagonists (8)
95
C. Other Adverse Effects
Dry mouth and thirst are common with many of these drugs
Tolvaptan may cause hypotension
Multiple adverse effects associated with lithium therapy have
been found and are discussed in CNS Drugs
Demeclocycline should be avoided in patients with liver
disease and in children younger than 12 years
Marc Imhotep Cray, M.D.
Renal Drugs Summary Table
Rosenfeld GC and Loose DS. Board Review Series
Pharmacology 6th ed. Philadelphia, PA:
Lippincott Williams & Wilkins, 2014.
Marc Imhotep Cray, M.D.
Case-based Discussions
97
Marc Imhotep Cray, M.D.
Case 7-Diuretics
98
A 64-year-old female with a past medical history of coronary artery disease,
hypertension, and congestive heart failure (CHF) presents with dyspnea at
rest and with exertion, orthopnea, and lower extremity pitting edema. Her
symptoms have worsened over the last 2 weeks and also include orthopnea,
worsening exercise tolerance, and tachypnea. On examination, she is
notably dyspneic and tachypneic, and also has jugular venous distension,
2+pitting edema, and rales on lung examination.
Patient is also found to have an audible S3. Her chest x-ray, pro-Brain
Natriuretic Peptide (BNP) level, and echocardiogram confirm the clinical
suspicion of CHF exacerbation with pulmonary edema. She is already on
maximal medical therapy with an ACE inhibitor, beta blocker, statin, and
aspirin. She is appropriately placed on oxygen and given intravenous
furosemide.
_ What is the mechanism of action of furosemide?
_ What electrolyte abnormalities can be caused by furosemide?
Marc Imhotep Cray, M.D.
Summary:
99
A 64-year-old woman with pulmonary edema is prescribed
furosemide.
• Mechanism of action of furosemide: Inhibit active NaCl
reabsorption in the ascending limb of the loop of Henle,
increasing water and electrolyte excretion.
• Potential electrolyte abnormalities: Hypokalemia,
hypomagnesemia, and metabolic alkalosis because of enhanced
H + excretion.
Marc Imhotep Cray, M.D.
Clinical Correlation
100
 Loop diuretics given intravenously promote diuresis within minutes,
making them ideal for the treatment of acute pulmonary edema.
 Furosemide is the prototype and most widely used drug in this class.
 Loop diuretics inhibit NaCl reabsorption in the ascending limb of the loop
of Henle. This causes a marked increase in the excretion of both water
and electrolytes.
 The excretion of potassium, magnesium, and calcium ions are all
increased, which may cause clinically significant adverse effects.
 A metabolic alkalosis may also occur as a result of the excretion of
hydrogen ions.
o However, the ability to cause excretion of these electrolytes may also
provide a clinical benefit in certain situations.
o Forced diuresis by giving IV saline and furosemide is a primary
method of treatment of hypercalcemia.
Marc Imhotep Cray, M.D.
Case 8-Nondiuretic Inhibitors of Tubular Transport
101
Following his third episode of gouty arthritis, a 50-year-old man sees you in
the clinic. Each case was successfully treated acutely; however, your patient
is interested in trying to prevent future episodes. He is not on regular
medications and has a normal physical examination today. Blood work
reveals an elevated serum uric acid level and otherwise normal renal
function and electrolytes. A 24-hour urine collection for uric acid reveals
that he is under-excreting uric acid. Suspecting that this is the cause of his
recurrent gout, you place him on probenecid.
_ What is the mechanism of action of probenecid?
_ Which drugs could have their excretion inhibited by probenecid?
Marc Imhotep Cray, M.D.
Summary:
102
A 50-year-old man with recurrent gout is prescribed probenecid.
• Mechanism of action of probenecid: Inhibits secretion of
organic acids and decreases reabsorption of uric acid, causing a
net increase in secretion.
• Other drugs whose secretion could be inhibited: Penicillin,
indomethacin, and methotrexate.
Marc Imhotep Cray, M.D.
Clinical Correlation
103
 Gout is a disease in which uric acid crystals deposit in joints, causing an
extremely painful acute inflammatory arthritis.
 Persons with recurrent gout often have chronically elevated levels of uric
acid in their blood. This hyperuricemia is frequently caused by either
overproduction of uric acid or under-excretion of uric acid by the kidneys.
 Probenecid (and other uricosuric drugs) promotes the excretion of uric acid.
o It works by inhibiting the secretion of organic acids from the plasma into the tubular
lumen and blocking the reuptake of uric acid.
o The net result of this is an increase in the excretion of uric acid.
 The benefit of this is the prevention of recurrent gout attacks in chronic
under-excreters of uric acid.
 In those individuals who overproduce uric acid, allopurinol or febuxostat is
used.
o These drugs inhibit xanthine oxidase, a key enzyme in the production of uric acid.
o For patients with severe gout refractory to the above drugs, IV infusion of pegloticase
can quickly reduce serum urate and reduce deposits in joints.
Marc Imhotep Cray, M.D.
Practice Questions &
Answers/Explanations
104
Marc Imhotep Cray, M.D.
Question 1
A patient taking an oral diuretic for about 6 months presents with
elevated fasting and postprandial blood glucose levels. You check
the patient’s HbA1c and find it is elevated compared with normal
baseline values obtained 6 months ago. You suspect the glycemic
problems are diuretic-induced. What was the most likely cause?
a. Acetazolamide
b. Amiloride
c. Chlorothiazide
d. Spironolactone
e. Triamterene
91
Marc Imhotep Cray, M.D.
Answer 1
The answer is c. Thiazides and thiazide-like diuretics (eg,
chlorthalidone, metolazone) tend to elevate blood glucose levels,
impair glucose tolerance, and cause frank hyperglycemia.
Several mechanisms have been proposed to explain the effect:
insulin resistance is the most likely mechanism.
Elevations of blood glucose levels, or other manifestations of
glycemic control, are rarely associated with treatment with
acetazolamide (a), amiloride (b), spironolactone (d), or triamterene (e).
Refs. G&G, pp 686-690; Katzung, pp 260-261.
NB: You may recall that diazoxide (mainly used as a parenteral
drug for prompt lowering of blood pressure) can be used in its oral
dosage form to raise blood glucose levels in some hypoglycemic
states. It is, chemically, a thiazide, but is not used as a diuretic.) 92
Question 2
A patient with essential hypertension is being treated with hydrochlorothiazide and a calcium
channel blocker, and is doing well. He also takes atorvastatin for hypercholesterolemia, and
aspirin to reduce his risk of an acute coronary syndrome. He is now diagnosed with a seizure
disorder. We begin therapy with one of the suitable anticonvulsants that, fortunately, does
not alter the metabolism of any of the medications prescribed for his cardiovascular
problems. We’ve also learn that systemic administration of acetazolamide may prove to be a
useful adjunct to the anticonvulsant therapy: the metabolic acidosis it causes may help
suppress seizure development or spread. So, we start acetazolamide therapy too. What is the
most likely outcome of adding the acetazolamide?
a. Excessive rises of plasma sodium concentration
b. Hypertensive crisis (antagonism of both antihypertensive drugs)
c. Hypokalemia via synergistic actions with the thiazide
d. Spontaneous bleeding (potentiation of aspirin’s actions)
e. Sudden circulating volume expansion, onset of heart failure 93
Marc Imhotep Cray, M.D.
Answer 2
The answer is c. We seldom administer acetazolamide as a diuretic, because its
effects are “mild”; associated with significant changes of both urine pH (up) and
blood pH (down; metabolic acidosis); and self-limiting (once sufficient
bicarbonate has been lost from the blood, into the urine, refractoriness to
further diuresis occurs). More often we administer acetazolamide and other
carbonic anhydrase inhibitors for nonrenal/noncardiovascular problems, such
as to lower intraocular pressure in some cases of glaucoma (carbonic anhydrase
inhibitors inhibit aqueous humor formation) or as an adjunct to anticonvulsant
therapy as described here. As a result, we may forget that these systemically
administered drugs are diuretics, one common property of all the diuretics
being increased renal sodium loss (a natriuretic effect; thus, answer a is not
correct). We may even forget that carbonic anhydrase inhibitors, given
systemically, are potassium-wasting diuretics: they act proximally and deliver
extra sodium distally where, at the principal cells of the nephron, some extra
Na+ is taken up in exchange for additional K+ that gets eliminated in the urine.
94
Answer 2 cont.
In this scenario the patient is taking a thiazide, which is obviously potassium-wasting and
has potential in its own right to cause hypokalemia. Add a carbonic anhydrase to the regimen
and the risks of hypokalemia increase. Acetazolamide does not antagonize the
antihypertensive effects of thiazides or calcium channel blockers, nor provoke hypertension
or a hypertensive crisis (b). If there were any interactions between the acetazolamide and
the aspirin, it would be antagonism, not potentiation (d) of aspirin’s antiplatelet effects.
Aspirin undergoes renal tubular reabsorption, and that is a pH-dependent effect. Aspirin’s
reabsorption is reduced (that is, its excretion increases) in an alkaline urine, which is
precisely what occurs with acetazolamide. (You should recall that alkalinizing the urine is an
important adjunctive measure in treating severe salicylate poisoning, in part because it
reduces tubular reabsorption of salicylate.) There is no reason to suspect sudden rises of
blood volume, with or without concomitant heart failure from that (e). Indeed, the added
diuresis from the acetazolamide may, at least transiently, potentiate the effects of the
thiazide on urine volume, blood pressure, or both.
Refs. G&G, pp 677-681; Katzung, pp 256-257, 261-262, 265. 95
Marc Imhotep Cray, M.D.
Question 3
An elderly patient with a history of heart disease is brought to the
emergency room with difficulty breathing.
Examination reveals that she has pulmonary edema.
Which treatment is indicated?
A. Acetazolamide.
B. Chlorthalidone.
C. Furosemide.
D. Hydrochlorothiazide.
E. Spironolactone.
96
Marc Imhotep Cray, M.D.
Answer 3
Correct answer is C. This is a potentially fatal situation. It is
important to administer a diuretic that will reduce fluid
accumulation in the lungs and, thus, improve oxygenation
and heart function. The loop diuretics are most effective in
removing large fluid volumes from the body and are the
treatment of choice in this situation. In this situation, furosemide
should be administered intravenously. The other choices are
inappropriate.
97
Marc Imhotep Cray, M.D.
Question 4
A group of college students is planning a mountain climbing trip
to the Andes. Which would be appropriate for them to take to
prevent mountain sickness?
A. A thiazide diuretic such as hydrochlorothiazide.
B. An anticholinergic such as atropine.
C. A carbonic anhydrase inhibitor such as acetazolamide.
D. A loop diuretic such as furosemide.
E. A β-blocker such as metoprolol.
98
Marc Imhotep Cray, M.D.
Answer 4
Correct answer is C. Acetazolamide is used prophylactically
for several days before an ascent above 10,000 feet. This
treatment prevents the cerebral and pulmonary problems
associated with the syndrome as well as other difficulties,
such as nausea.
99
Marc Imhotep Cray, M.D.
Question 5
An alcoholic male has developed hepatic cirrhosis. To control
the ascites and edema, which should be prescribed?
A. Acetazolamide.
B. Chlorthalidone.
C. Furosemide.
D. Hydrochlorothiazide.
E. Spironolactone.
100
Marc Imhotep Cray, M.D.
Answer 5
Correct answer is E. Spironolactone is very effective in the
treatment of hepatic edema. These patients are frequently
resistant to the diuretic action of loop diuretics, although
a combination with spironolactone may be beneficial. The other
agents are not indicated.
101
Marc Imhotep Cray, M.D.
Question 6
A 55-year-old male with kidney stones has been placed on a
diuretic to decrease calcium excretion. However, after a few
weeks, he develops an attack of gout. Which diuretic was he
taking?
A. Furosemide.
B. Hydrochlorothiazide.
C. Spironolactone.
D. Triamterene.
E. Urea.
102
Marc Imhotep Cray, M.D.
Answer 6
Correct answer is B. Hydrochlorothiazide is effective in
increasing calcium reabsorption, thus decreasing the amount
of calcium excreted, and decreasing the formation of kidney
stones that contain calcium phosphate or calcium oxalate.
However, hydrochlorothiazide can also inhibit the excretion of
uric acid and cause its accumulation, leading to an attack of
gout in some individuals. Furosemide increases the excretion
of calcium, whereas the K+-sparing osmotic diuretics,
spironolactone and triamterene, and urea do not have an effect.
103
Marc Imhotep Cray, M.D.
Question 7
A 75-year-old woman with hypertension is being treated
with a thiazide. Her blood pressure responds well and
reads at 120/76 mm Hg. After several months on the
medication, she complains of being tired and weak. An
analysis of the blood indicates low values for which of
the following?
A. Calcium.
B. Glucose.
C. Potassium.
D. Sodium.
E. Uric acid.
104
Marc Imhotep Cray, M.D.
Answer 7
Correct answer is C. Hypokalemia is a common adverse
effect of the thiazides and causes fatigue and lethargy in the
patient. Supplementation with potassium chloride or foods
high in K+ corrects the problem. Alternatively, a potassium
sparing diuretic, such as spironolactone, may be added.
Calcium, uric acid, and glucose are usually elevated by thiazide
diuretics. Sodium loss would not weaken the patient.
105
Marc Imhotep Cray, M.D.
Question 8
Which is contraindicated in a patient with hyperkalemia?
A. Acetazolamide.
B. Chlorthalidone.
C. Chlorothiazide.
D. Ethacrynic acid.
E. Spironolactone.
106
Marc Imhotep Cray, M.D.
Answer 8
Correct answer is E. Spironolactone acts in the collecting tubule to
inhibit Na+ reabsorption and K+ excretion. It is extremely important
that patients who are treated with any potassium-sparing diuretic
be closely monitored for potassium levels. Exogenous potassium
supplementation is usually discontinued when potassium-sparing
diuretic therapy is instituted and spironolactone is contraindicated
in patients with hyperkalemia. The other drugs promote the
excretion of potassium.
107
Marc Imhotep Cray, M.D.
Question 9
Which of the following should be avoided in a patient with a
history of severe anaphylactic reaction to sulfa medications?
A. Amiloride.
B. Hydrochlorothiazide.
C. Mannitol.
D. Spironolactone.
E. Triamterene.
108
Marc Imhotep Cray, M.D.
Answer 9
Correct answer is B. Hydrochlorothiazide, like many thiazide
and thiazide-like diuretics, contains a sulfa moiety within its
chemical structure. It is important to avoid use in those individuals
with severe hypersensitivity to sulfa medications. It
may be used with caution, however, in those with only minor
reaction to sulfa medications.
109
Marc Imhotep Cray, M.D.
Question 10
A male patient is placed on a new medication and notes that his
breasts have become enlarged and tender to the touch. Which
medication is he most likely taking?
A. Chlorthalidone.
B. Furosemide.
C. Hydrochlorothiazide.
D. Spironolactone.
E. Triamterene.
110
Marc Imhotep Cray, M.D.
Answer 10
Correct answer = D. An adverse drug reaction to spironolactone is
gynecomastia due to its effects on androgens and progesterone in
the body. Eplerenone may be a suitable alternative if the patient is
in need of an aldosterone antagonist but has a history of
gynecomastia.
111
Marc Imhotep Cray, M.D.
Question 11
A patient presents to the emergency department with an extreme
headache. After a thorough workup, the attending physician
concludes that the pain is due to increased intracranial pressure.
Which diuretic would work best to reduce this pressure?
A. Acetazolamide.
B. Indapamide.
C. Furosemide.
D. Hydrochlorothiazide.
E. Mannitol.
112
Marc Imhotep Cray, M.D.
Answer 11
Correct answer = E. Osmotic diuretics, such as mannitol, are a
mainstay of treatment for patients with increased intracranial
pressure or acute renal failure due to shock, drug toxicities, and
trauma.
113
Marc Imhotep Cray, M.D.
Question 12
Which diuretic has been shown to improve blood pressure in
resistant hypertension or those already treated with three blood
pressure medications including a thiazide or thiazide-like diuretic?
A. Chlorthalidone.
B. Indapamide.
C. Furosemide.
D. Mannitol.
E. Spironolactone.
114
Marc Imhotep Cray, M.D.
Answer 12
Correct answer = E. Resistant hypertension, defined by
the use of three or more medications without reaching the
blood pressure goal, often responds well to aldosterone
antagonists. This effect can be seen in those with or without
elevated aldosterone levels.
115
Marc Imhotep Cray, M.D.
See next slide for sources and links to additional study tools and resources.
130
Marc Imhotep Cray, M.D.
Sources and further study:
eLearning
Renal cloud folder tools and resources
MedPharm Guidebook:
Unit 9 Drugs Used to Affect Renal Function
Renal Pharmacology eNotes
Clinical Pharmacology Cases 7, 8, & 55 (Learning Triggers)
Textbooks
Brunton LL, Chabner BA , Knollmann BC (Eds.). Goodman and Gilman’s The Pharmacological
Basis of Therapeutics. 12th ed. New York: McGraw-Hill, 2011
Katzung, Masters, Trevor. Basic and Clinical Pharmacology, 12th ed. New York: McGraw-Hill,
2012
Mulroney SE. and Myers AK. Netter's Essential Physiology. Philadelphia: Saunders, 2009
Raff RB, Rawls SM, Beyzarov EP. Netter's Illustrated Pharmacology, Updated Edition.
Philadelphia: Sanders, 2014
Toy E C. et.al. Case Files-Pharmacology Lange 3rd ed. New York: McGraw-Hill 2014.
131

More Related Content

What's hot

Pharmacology of Diuretics
Pharmacology of DiureticsPharmacology of Diuretics
Pharmacology of Diuretics
Koppala RVS Chaitanya
 
Sedatives and hypnotics
Sedatives and hypnoticsSedatives and hypnotics
Sedatives and hypnotics
SONALPANDE5
 
Pharmacology angina
Pharmacology   anginaPharmacology   angina
Pharmacology anginaMBBS IMS MSU
 
3.Diuretics
3.Diuretics3.Diuretics
3.Diuretics
Mirza Anwar Baig
 
skeletal muscle relaxants.pptx
skeletal muscle relaxants.pptxskeletal muscle relaxants.pptx
skeletal muscle relaxants.pptx
Vijay Salvekar
 
Diuretics
DiureticsDiuretics
Diuretics
Sujit Karpe
 
Warfarin
WarfarinWarfarin
Warfarin
Darya Daoud
 
Pharmacology Kidney Drugs
Pharmacology   Kidney DrugsPharmacology   Kidney Drugs
Pharmacology Kidney Drugs
pinoy nurze
 
Anti-Parkinsonism Drugs / Drugs Used in the treatment of Parkinson's Disease
Anti-Parkinsonism Drugs / Drugs Used in the treatment of Parkinson's DiseaseAnti-Parkinsonism Drugs / Drugs Used in the treatment of Parkinson's Disease
Anti-Parkinsonism Drugs / Drugs Used in the treatment of Parkinson's Disease
Kameshwaran Sugavanam
 
Drugs acting on the kidney lectures 1 and 2
Drugs acting on the kidney lectures 1 and 2Drugs acting on the kidney lectures 1 and 2
Drugs acting on the kidney lectures 1 and 2
Pharmacology Education Project
 
Antiparkinsonian drugs
Antiparkinsonian drugsAntiparkinsonian drugs
Antiparkinsonian drugs
Dr.Arka Mondal
 
Sedative and hypnotics
Sedative and hypnoticsSedative and hypnotics
Sedative and hypnotics
Shagufta Farooqui
 
Antiparkinsonian Drugs (Full Lecture)
Antiparkinsonian Drugs (Full Lecture) Antiparkinsonian Drugs (Full Lecture)
Antiparkinsonian Drugs (Full Lecture)
Sawsan Aboul-Fotouh
 
Diuretics
DiureticsDiuretics
Diuretics
Dr. HN Singh
 
Diuretics
DiureticsDiuretics
Diuretics
Reza Heidari
 
Expectorant and antitussives
Expectorant and antitussivesExpectorant and antitussives
Expectorant and antitussives
SnehalChakorkar
 
Proton pump inhibitor
Proton pump inhibitorProton pump inhibitor
Proton pump inhibitor
Asiful alam
 
ACE inhibitors drugs
ACE inhibitors drugsACE inhibitors drugs
ACE inhibitors drugs
Irresolute Tanvir
 
Antidiuretic
AntidiureticAntidiuretic
Antidiuretic
masum1416812
 

What's hot (20)

Pharmacology of Diuretics
Pharmacology of DiureticsPharmacology of Diuretics
Pharmacology of Diuretics
 
Sedatives and hypnotics
Sedatives and hypnoticsSedatives and hypnotics
Sedatives and hypnotics
 
Pharmacology angina
Pharmacology   anginaPharmacology   angina
Pharmacology angina
 
Cardiac glycosides
Cardiac glycosidesCardiac glycosides
Cardiac glycosides
 
3.Diuretics
3.Diuretics3.Diuretics
3.Diuretics
 
skeletal muscle relaxants.pptx
skeletal muscle relaxants.pptxskeletal muscle relaxants.pptx
skeletal muscle relaxants.pptx
 
Diuretics
DiureticsDiuretics
Diuretics
 
Warfarin
WarfarinWarfarin
Warfarin
 
Pharmacology Kidney Drugs
Pharmacology   Kidney DrugsPharmacology   Kidney Drugs
Pharmacology Kidney Drugs
 
Anti-Parkinsonism Drugs / Drugs Used in the treatment of Parkinson's Disease
Anti-Parkinsonism Drugs / Drugs Used in the treatment of Parkinson's DiseaseAnti-Parkinsonism Drugs / Drugs Used in the treatment of Parkinson's Disease
Anti-Parkinsonism Drugs / Drugs Used in the treatment of Parkinson's Disease
 
Drugs acting on the kidney lectures 1 and 2
Drugs acting on the kidney lectures 1 and 2Drugs acting on the kidney lectures 1 and 2
Drugs acting on the kidney lectures 1 and 2
 
Antiparkinsonian drugs
Antiparkinsonian drugsAntiparkinsonian drugs
Antiparkinsonian drugs
 
Sedative and hypnotics
Sedative and hypnoticsSedative and hypnotics
Sedative and hypnotics
 
Antiparkinsonian Drugs (Full Lecture)
Antiparkinsonian Drugs (Full Lecture) Antiparkinsonian Drugs (Full Lecture)
Antiparkinsonian Drugs (Full Lecture)
 
Diuretics
DiureticsDiuretics
Diuretics
 
Diuretics
DiureticsDiuretics
Diuretics
 
Expectorant and antitussives
Expectorant and antitussivesExpectorant and antitussives
Expectorant and antitussives
 
Proton pump inhibitor
Proton pump inhibitorProton pump inhibitor
Proton pump inhibitor
 
ACE inhibitors drugs
ACE inhibitors drugsACE inhibitors drugs
ACE inhibitors drugs
 
Antidiuretic
AntidiureticAntidiuretic
Antidiuretic
 

Viewers also liked

Pharmacology KDT problem based questions
Pharmacology KDT problem based questionsPharmacology KDT problem based questions
Pharmacology KDT problem based questions
Abhinav Kumar
 
Urinary System Drugs
Urinary System DrugsUrinary System Drugs
Urinary System Drugs
guest9bc2b8
 
Cardiac electrophysiology and pharmacology - drdhriti
Cardiac electrophysiology and pharmacology - drdhritiCardiac electrophysiology and pharmacology - drdhriti
Cardiac electrophysiology and pharmacology - drdhriti
http://neigrihms.gov.in/
 
NurseReview.Org Pharmacology - Kidney Drugs
NurseReview.Org Pharmacology - Kidney DrugsNurseReview.Org Pharmacology - Kidney Drugs
NurseReview.Org Pharmacology - Kidney Drugs
Nurse ReviewDotOrg
 
Drug use in hepatic and renal impairment
Drug use in hepatic and renal impairmentDrug use in hepatic and renal impairment
Drug use in hepatic and renal impairment
Akshil Mehta
 
Sedative hypnotics.ppt - dr dhriti
Sedative hypnotics.ppt - dr dhriti Sedative hypnotics.ppt - dr dhriti
Sedative hypnotics.ppt - dr dhriti
http://neigrihms.gov.in/
 
Aminoglycosides(medicinal chemistry by p.ravisankar)
Aminoglycosides(medicinal chemistry by p.ravisankar)Aminoglycosides(medicinal chemistry by p.ravisankar)
Aminoglycosides(medicinal chemistry by p.ravisankar)
Dr. Ravi Sankar
 
Problem based learning
Problem based learningProblem based learning
Problem based learning
Soumya Sahoo
 

Viewers also liked (9)

urinary system drugs
 urinary system drugs urinary system drugs
urinary system drugs
 
Pharmacology KDT problem based questions
Pharmacology KDT problem based questionsPharmacology KDT problem based questions
Pharmacology KDT problem based questions
 
Urinary System Drugs
Urinary System DrugsUrinary System Drugs
Urinary System Drugs
 
Cardiac electrophysiology and pharmacology - drdhriti
Cardiac electrophysiology and pharmacology - drdhritiCardiac electrophysiology and pharmacology - drdhriti
Cardiac electrophysiology and pharmacology - drdhriti
 
NurseReview.Org Pharmacology - Kidney Drugs
NurseReview.Org Pharmacology - Kidney DrugsNurseReview.Org Pharmacology - Kidney Drugs
NurseReview.Org Pharmacology - Kidney Drugs
 
Drug use in hepatic and renal impairment
Drug use in hepatic and renal impairmentDrug use in hepatic and renal impairment
Drug use in hepatic and renal impairment
 
Sedative hypnotics.ppt - dr dhriti
Sedative hypnotics.ppt - dr dhriti Sedative hypnotics.ppt - dr dhriti
Sedative hypnotics.ppt - dr dhriti
 
Aminoglycosides(medicinal chemistry by p.ravisankar)
Aminoglycosides(medicinal chemistry by p.ravisankar)Aminoglycosides(medicinal chemistry by p.ravisankar)
Aminoglycosides(medicinal chemistry by p.ravisankar)
 
Problem based learning
Problem based learningProblem based learning
Problem based learning
 

Similar to Clinical Pharmacology of Drugs Used to Affect Renal Function

Basic Pharmacology of Diuretics
Basic Pharmacology of DiureticsBasic Pharmacology of Diuretics
Basic Pharmacology of Diuretics
Imhotep Virtual Medical School
 
Renal Physiology and Regulation of Water and Inorganic Ions
Renal Physiology and Regulation of Water and Inorganic IonsRenal Physiology and Regulation of Water and Inorganic Ions
Renal Physiology and Regulation of Water and Inorganic Ions
Imhotep Virtual Medical School
 
Renal system
Renal systemRenal system
Renal system
Pharmtechfau
 
Diuretics-Antiemetics-ADRs-MM-pharm-weekend-15.03-Alex-Conway.pptx
Diuretics-Antiemetics-ADRs-MM-pharm-weekend-15.03-Alex-Conway.pptxDiuretics-Antiemetics-ADRs-MM-pharm-weekend-15.03-Alex-Conway.pptx
Diuretics-Antiemetics-ADRs-MM-pharm-weekend-15.03-Alex-Conway.pptx
AnimeWizard
 
Souvik diueretics note ppt
Souvik diueretics note pptSouvik diueretics note ppt
Souvik diueretics note ppt
souvik388
 
Renal Insufficiency and Dialysis, Urinary Incontinence and Urinary Tract Calculi
Renal Insufficiency and Dialysis, Urinary Incontinence and Urinary Tract CalculiRenal Insufficiency and Dialysis, Urinary Incontinence and Urinary Tract Calculi
Renal Insufficiency and Dialysis, Urinary Incontinence and Urinary Tract Calculi
Imhotep Virtual Medical School
 
International Journal of Mathematics and Statistics Invention (IJMSI)
International Journal of Mathematics and Statistics Invention (IJMSI) International Journal of Mathematics and Statistics Invention (IJMSI)
International Journal of Mathematics and Statistics Invention (IJMSI)
inventionjournals
 
CRRT by Ashraf Qotmosh.pdf
CRRT by Ashraf Qotmosh.pdfCRRT by Ashraf Qotmosh.pdf
CRRT by Ashraf Qotmosh.pdf
RAGHAVENDRAVAGYANNAV
 
Diuretics
DiureticsDiuretics
Diuretics manik
Diuretics manikDiuretics manik
Diuretics manik
Imran Nur Manik
 
Perioperative fluid & electrolytes Therapy - part 2
Perioperative fluid & electrolytes Therapy - part 2Perioperative fluid & electrolytes Therapy - part 2
Perioperative fluid & electrolytes Therapy - part 2
mansoor masjedi
 
Pharmacology of diuretics-antidiuretics
Pharmacology of diuretics-antidiureticsPharmacology of diuretics-antidiuretics
Pharmacology of diuretics-antidiuretics
Kola Phani Kumar
 
Lecture 1 adithan diuretics july 22, 2016 mgmcri
Lecture 1 adithan diuretics july 22, 2016 mgmcriLecture 1 adithan diuretics july 22, 2016 mgmcri
Lecture 1 adithan diuretics july 22, 2016 mgmcri
Mahatma Gandhi Medical College & Hospital
 
Diuretics new
Diuretics newDiuretics new
Diuretics by P.Ravisankar
Diuretics by P.RavisankarDiuretics by P.Ravisankar
Diuretics by P.Ravisankar
Dr. Ravi Sankar
 
Hepatic and Renal Clearance.pptx
Hepatic and Renal Clearance.pptxHepatic and Renal Clearance.pptx
Hepatic and Renal Clearance.pptx
AbuHumaid1
 
PHYSIOLOGY OF CONNECTING TUBULE AND COLLECTING DUCT
PHYSIOLOGY OF CONNECTING TUBULE AND COLLECTING DUCTPHYSIOLOGY OF CONNECTING TUBULE AND COLLECTING DUCT
PHYSIOLOGY OF CONNECTING TUBULE AND COLLECTING DUCT
Ahad Lodhi
 
Electrolyte imbalance
Electrolyte imbalanceElectrolyte imbalance
Electrolyte imbalance
AbdelrahmanAbdelkade7
 

Similar to Clinical Pharmacology of Drugs Used to Affect Renal Function (20)

Basic Pharmacology of Diuretics
Basic Pharmacology of DiureticsBasic Pharmacology of Diuretics
Basic Pharmacology of Diuretics
 
Renal Physiology and Regulation of Water and Inorganic Ions
Renal Physiology and Regulation of Water and Inorganic IonsRenal Physiology and Regulation of Water and Inorganic Ions
Renal Physiology and Regulation of Water and Inorganic Ions
 
Renal system
Renal systemRenal system
Renal system
 
Diuretics-Antiemetics-ADRs-MM-pharm-weekend-15.03-Alex-Conway.pptx
Diuretics-Antiemetics-ADRs-MM-pharm-weekend-15.03-Alex-Conway.pptxDiuretics-Antiemetics-ADRs-MM-pharm-weekend-15.03-Alex-Conway.pptx
Diuretics-Antiemetics-ADRs-MM-pharm-weekend-15.03-Alex-Conway.pptx
 
Souvik diueretics note ppt
Souvik diueretics note pptSouvik diueretics note ppt
Souvik diueretics note ppt
 
Renal Insufficiency and Dialysis, Urinary Incontinence and Urinary Tract Calculi
Renal Insufficiency and Dialysis, Urinary Incontinence and Urinary Tract CalculiRenal Insufficiency and Dialysis, Urinary Incontinence and Urinary Tract Calculi
Renal Insufficiency and Dialysis, Urinary Incontinence and Urinary Tract Calculi
 
Diuretics
DiureticsDiuretics
Diuretics
 
Diuretics
DiureticsDiuretics
Diuretics
 
International Journal of Mathematics and Statistics Invention (IJMSI)
International Journal of Mathematics and Statistics Invention (IJMSI) International Journal of Mathematics and Statistics Invention (IJMSI)
International Journal of Mathematics and Statistics Invention (IJMSI)
 
CRRT by Ashraf Qotmosh.pdf
CRRT by Ashraf Qotmosh.pdfCRRT by Ashraf Qotmosh.pdf
CRRT by Ashraf Qotmosh.pdf
 
Diuretics
DiureticsDiuretics
Diuretics
 
Diuretics manik
Diuretics manikDiuretics manik
Diuretics manik
 
Perioperative fluid & electrolytes Therapy - part 2
Perioperative fluid & electrolytes Therapy - part 2Perioperative fluid & electrolytes Therapy - part 2
Perioperative fluid & electrolytes Therapy - part 2
 
Pharmacology of diuretics-antidiuretics
Pharmacology of diuretics-antidiureticsPharmacology of diuretics-antidiuretics
Pharmacology of diuretics-antidiuretics
 
Lecture 1 adithan diuretics july 22, 2016 mgmcri
Lecture 1 adithan diuretics july 22, 2016 mgmcriLecture 1 adithan diuretics july 22, 2016 mgmcri
Lecture 1 adithan diuretics july 22, 2016 mgmcri
 
Diuretics new
Diuretics newDiuretics new
Diuretics new
 
Diuretics by P.Ravisankar
Diuretics by P.RavisankarDiuretics by P.Ravisankar
Diuretics by P.Ravisankar
 
Hepatic and Renal Clearance.pptx
Hepatic and Renal Clearance.pptxHepatic and Renal Clearance.pptx
Hepatic and Renal Clearance.pptx
 
PHYSIOLOGY OF CONNECTING TUBULE AND COLLECTING DUCT
PHYSIOLOGY OF CONNECTING TUBULE AND COLLECTING DUCTPHYSIOLOGY OF CONNECTING TUBULE AND COLLECTING DUCT
PHYSIOLOGY OF CONNECTING TUBULE AND COLLECTING DUCT
 
Electrolyte imbalance
Electrolyte imbalanceElectrolyte imbalance
Electrolyte imbalance
 

More from Imhotep Virtual Medical School

Oncologic Pathology_A Case-based Organ Systems Review (USMLE Step 1)
Oncologic Pathology_A Case-based Organ Systems Review (USMLE Step 1)Oncologic Pathology_A Case-based Organ Systems Review (USMLE Step 1)
Oncologic Pathology_A Case-based Organ Systems Review (USMLE Step 1)
Imhotep Virtual Medical School
 
Pathology and Pathophysiology of Shock
Pathology and Pathophysiology of ShockPathology and Pathophysiology of Shock
Pathology and Pathophysiology of Shock
Imhotep Virtual Medical School
 
Drugs Used In Disorders of the Reproductive System
Drugs Used In Disorders of the Reproductive SystemDrugs Used In Disorders of the Reproductive System
Drugs Used In Disorders of the Reproductive System
Imhotep Virtual Medical School
 
Reproductive System Pathology_FM Breast and FM Reproductive Systems
Reproductive System Pathology_FM Breast and FM Reproductive SystemsReproductive System Pathology_FM Breast and FM Reproductive Systems
Reproductive System Pathology_FM Breast and FM Reproductive Systems
Imhotep Virtual Medical School
 
Reproductive System Pathology_Male Reproductive Systems
Reproductive System Pathology_Male Reproductive SystemsReproductive System Pathology_Male Reproductive Systems
Reproductive System Pathology_Male Reproductive Systems
Imhotep Virtual Medical School
 
Nervous System Pathology_A Case-based Learning Approach
Nervous System Pathology_A Case-based Learning ApproachNervous System Pathology_A Case-based Learning Approach
Nervous System Pathology_A Case-based Learning Approach
Imhotep Virtual Medical School
 
CVS Function, Regulation of the Heart and Overview of Therapeutic Goals in CV...
CVS Function, Regulation of the Heart and Overview of Therapeutic Goals in CV...CVS Function, Regulation of the Heart and Overview of Therapeutic Goals in CV...
CVS Function, Regulation of the Heart and Overview of Therapeutic Goals in CV...
Imhotep Virtual Medical School
 
Cardiovascular Pathology Case-based_Gross and Microscopic
Cardiovascular Pathology Case-based_Gross and MicroscopicCardiovascular Pathology Case-based_Gross and Microscopic
Cardiovascular Pathology Case-based_Gross and Microscopic
Imhotep Virtual Medical School
 
HIV / AIDS Pathology
HIV / AIDS PathologyHIV / AIDS Pathology
HIV / AIDS Pathology
Imhotep Virtual Medical School
 
Sepsis & Septic Shock
Sepsis & Septic ShockSepsis & Septic Shock
Sepsis & Septic Shock
Imhotep Virtual Medical School
 
Drugs Used in infectious Disease_Antibiotics
Drugs Used in infectious Disease_AntibioticsDrugs Used in infectious Disease_Antibiotics
Drugs Used in infectious Disease_Antibiotics
Imhotep Virtual Medical School
 
Hematopoietic and Lymphoid Systems Pathology
Hematopoietic and Lymphoid Systems  PathologyHematopoietic and Lymphoid Systems  Pathology
Hematopoietic and Lymphoid Systems Pathology
Imhotep Virtual Medical School
 
Drugs Used in Neoplastic Disorders
Drugs Used in Neoplastic DisordersDrugs Used in Neoplastic Disorders
Drugs Used in Neoplastic Disorders
Imhotep Virtual Medical School
 
Neoplasia & Oncologic Pathology
Neoplasia & Oncologic PathologyNeoplasia & Oncologic Pathology
Neoplasia & Oncologic Pathology
Imhotep Virtual Medical School
 
Clinical Pharmacology for Medical Students_USMLE Step 1 & 2 Review
Clinical Pharmacology for Medical Students_USMLE Step 1 & 2 ReviewClinical Pharmacology for Medical Students_USMLE Step 1 & 2 Review
Clinical Pharmacology for Medical Students_USMLE Step 1 & 2 Review
Imhotep Virtual Medical School
 
Make the Dx_ A Case-based Intro to Select Cardiovascular and Respiratory Dise...
Make the Dx_ A Case-based Intro to Select Cardiovascular and Respiratory Dise...Make the Dx_ A Case-based Intro to Select Cardiovascular and Respiratory Dise...
Make the Dx_ A Case-based Intro to Select Cardiovascular and Respiratory Dise...
Imhotep Virtual Medical School
 
Myocardial infarction_ Causes, Symptoms, Diagnosis, Treatment, and Pathology
Myocardial infarction_ Causes, Symptoms, Diagnosis, Treatment, and PathologyMyocardial infarction_ Causes, Symptoms, Diagnosis, Treatment, and Pathology
Myocardial infarction_ Causes, Symptoms, Diagnosis, Treatment, and Pathology
Imhotep Virtual Medical School
 
Basic CXR Interpretation_Diagnostic Radiographs
Basic CXR Interpretation_Diagnostic RadiographsBasic CXR Interpretation_Diagnostic Radiographs
Basic CXR Interpretation_Diagnostic Radiographs
Imhotep Virtual Medical School
 
Electrocardiogram (ECG) Interpretation_Module 1 of 2
Electrocardiogram (ECG) Interpretation_Module 1 of 2Electrocardiogram (ECG) Interpretation_Module 1 of 2
Electrocardiogram (ECG) Interpretation_Module 1 of 2
Imhotep Virtual Medical School
 
Autonomic Nervous System Physiology and Pharmacology_Overview| Review of ANS
Autonomic Nervous System Physiology and Pharmacology_Overview| Review of ANSAutonomic Nervous System Physiology and Pharmacology_Overview| Review of ANS
Autonomic Nervous System Physiology and Pharmacology_Overview| Review of ANS
Imhotep Virtual Medical School
 

More from Imhotep Virtual Medical School (20)

Oncologic Pathology_A Case-based Organ Systems Review (USMLE Step 1)
Oncologic Pathology_A Case-based Organ Systems Review (USMLE Step 1)Oncologic Pathology_A Case-based Organ Systems Review (USMLE Step 1)
Oncologic Pathology_A Case-based Organ Systems Review (USMLE Step 1)
 
Pathology and Pathophysiology of Shock
Pathology and Pathophysiology of ShockPathology and Pathophysiology of Shock
Pathology and Pathophysiology of Shock
 
Drugs Used In Disorders of the Reproductive System
Drugs Used In Disorders of the Reproductive SystemDrugs Used In Disorders of the Reproductive System
Drugs Used In Disorders of the Reproductive System
 
Reproductive System Pathology_FM Breast and FM Reproductive Systems
Reproductive System Pathology_FM Breast and FM Reproductive SystemsReproductive System Pathology_FM Breast and FM Reproductive Systems
Reproductive System Pathology_FM Breast and FM Reproductive Systems
 
Reproductive System Pathology_Male Reproductive Systems
Reproductive System Pathology_Male Reproductive SystemsReproductive System Pathology_Male Reproductive Systems
Reproductive System Pathology_Male Reproductive Systems
 
Nervous System Pathology_A Case-based Learning Approach
Nervous System Pathology_A Case-based Learning ApproachNervous System Pathology_A Case-based Learning Approach
Nervous System Pathology_A Case-based Learning Approach
 
CVS Function, Regulation of the Heart and Overview of Therapeutic Goals in CV...
CVS Function, Regulation of the Heart and Overview of Therapeutic Goals in CV...CVS Function, Regulation of the Heart and Overview of Therapeutic Goals in CV...
CVS Function, Regulation of the Heart and Overview of Therapeutic Goals in CV...
 
Cardiovascular Pathology Case-based_Gross and Microscopic
Cardiovascular Pathology Case-based_Gross and MicroscopicCardiovascular Pathology Case-based_Gross and Microscopic
Cardiovascular Pathology Case-based_Gross and Microscopic
 
HIV / AIDS Pathology
HIV / AIDS PathologyHIV / AIDS Pathology
HIV / AIDS Pathology
 
Sepsis & Septic Shock
Sepsis & Septic ShockSepsis & Septic Shock
Sepsis & Septic Shock
 
Drugs Used in infectious Disease_Antibiotics
Drugs Used in infectious Disease_AntibioticsDrugs Used in infectious Disease_Antibiotics
Drugs Used in infectious Disease_Antibiotics
 
Hematopoietic and Lymphoid Systems Pathology
Hematopoietic and Lymphoid Systems  PathologyHematopoietic and Lymphoid Systems  Pathology
Hematopoietic and Lymphoid Systems Pathology
 
Drugs Used in Neoplastic Disorders
Drugs Used in Neoplastic DisordersDrugs Used in Neoplastic Disorders
Drugs Used in Neoplastic Disorders
 
Neoplasia & Oncologic Pathology
Neoplasia & Oncologic PathologyNeoplasia & Oncologic Pathology
Neoplasia & Oncologic Pathology
 
Clinical Pharmacology for Medical Students_USMLE Step 1 & 2 Review
Clinical Pharmacology for Medical Students_USMLE Step 1 & 2 ReviewClinical Pharmacology for Medical Students_USMLE Step 1 & 2 Review
Clinical Pharmacology for Medical Students_USMLE Step 1 & 2 Review
 
Make the Dx_ A Case-based Intro to Select Cardiovascular and Respiratory Dise...
Make the Dx_ A Case-based Intro to Select Cardiovascular and Respiratory Dise...Make the Dx_ A Case-based Intro to Select Cardiovascular and Respiratory Dise...
Make the Dx_ A Case-based Intro to Select Cardiovascular and Respiratory Dise...
 
Myocardial infarction_ Causes, Symptoms, Diagnosis, Treatment, and Pathology
Myocardial infarction_ Causes, Symptoms, Diagnosis, Treatment, and PathologyMyocardial infarction_ Causes, Symptoms, Diagnosis, Treatment, and Pathology
Myocardial infarction_ Causes, Symptoms, Diagnosis, Treatment, and Pathology
 
Basic CXR Interpretation_Diagnostic Radiographs
Basic CXR Interpretation_Diagnostic RadiographsBasic CXR Interpretation_Diagnostic Radiographs
Basic CXR Interpretation_Diagnostic Radiographs
 
Electrocardiogram (ECG) Interpretation_Module 1 of 2
Electrocardiogram (ECG) Interpretation_Module 1 of 2Electrocardiogram (ECG) Interpretation_Module 1 of 2
Electrocardiogram (ECG) Interpretation_Module 1 of 2
 
Autonomic Nervous System Physiology and Pharmacology_Overview| Review of ANS
Autonomic Nervous System Physiology and Pharmacology_Overview| Review of ANSAutonomic Nervous System Physiology and Pharmacology_Overview| Review of ANS
Autonomic Nervous System Physiology and Pharmacology_Overview| Review of ANS
 

Recently uploaded

Best Ayurvedic medicine for Gas and Indigestion
Best Ayurvedic medicine for Gas and IndigestionBest Ayurvedic medicine for Gas and Indigestion
Best Ayurvedic medicine for Gas and Indigestion
Swastik Ayurveda
 
263778731218 Abortion Clinic /Pills In Harare ,
263778731218 Abortion Clinic /Pills In Harare ,263778731218 Abortion Clinic /Pills In Harare ,
263778731218 Abortion Clinic /Pills In Harare ,
sisternakatoto
 
micro teaching on communication m.sc nursing.pdf
micro teaching on communication m.sc nursing.pdfmicro teaching on communication m.sc nursing.pdf
micro teaching on communication m.sc nursing.pdf
Anurag Sharma
 
Effective-Soaps-for-Fungal-Skin-Infections.pptx
Effective-Soaps-for-Fungal-Skin-Infections.pptxEffective-Soaps-for-Fungal-Skin-Infections.pptx
Effective-Soaps-for-Fungal-Skin-Infections.pptx
SwisschemDerma
 
Colonic and anorectal physiology with surgical implications
Colonic and anorectal physiology with surgical implicationsColonic and anorectal physiology with surgical implications
Colonic and anorectal physiology with surgical implications
Dr Maria Tamanna
 
Ophthalmology Clinical Tests for OSCE exam
Ophthalmology Clinical Tests for OSCE examOphthalmology Clinical Tests for OSCE exam
Ophthalmology Clinical Tests for OSCE exam
KafrELShiekh University
 
Ozempic: Preoperative Management of Patients on GLP-1 Receptor Agonists
Ozempic: Preoperative Management of Patients on GLP-1 Receptor Agonists  Ozempic: Preoperative Management of Patients on GLP-1 Receptor Agonists
Ozempic: Preoperative Management of Patients on GLP-1 Receptor Agonists
Saeid Safari
 
A Classical Text Review on Basavarajeeyam
A Classical Text Review on BasavarajeeyamA Classical Text Review on Basavarajeeyam
A Classical Text Review on Basavarajeeyam
Dr. Jyothirmai Paindla
 
SURGICAL ANATOMY OF THE RETROPERITONEUM, ADRENALS, KIDNEYS AND URETERS.pptx
SURGICAL ANATOMY OF THE RETROPERITONEUM, ADRENALS, KIDNEYS AND URETERS.pptxSURGICAL ANATOMY OF THE RETROPERITONEUM, ADRENALS, KIDNEYS AND URETERS.pptx
SURGICAL ANATOMY OF THE RETROPERITONEUM, ADRENALS, KIDNEYS AND URETERS.pptx
Bright Chipili
 
Dehradun #ℂall #gIRLS Oyo Hotel 8107221448 #ℂall #gIRL in Dehradun
Dehradun #ℂall #gIRLS Oyo Hotel 8107221448 #ℂall #gIRL in DehradunDehradun #ℂall #gIRLS Oyo Hotel 8107221448 #ℂall #gIRL in Dehradun
Dehradun #ℂall #gIRLS Oyo Hotel 8107221448 #ℂall #gIRL in Dehradun
chandankumarsmartiso
 
Thyroid Gland- Gross Anatomy by Dr. Rabia Inam Gandapore.pptx
Thyroid Gland- Gross Anatomy by Dr. Rabia Inam Gandapore.pptxThyroid Gland- Gross Anatomy by Dr. Rabia Inam Gandapore.pptx
Thyroid Gland- Gross Anatomy by Dr. Rabia Inam Gandapore.pptx
Dr. Rabia Inam Gandapore
 
Physiology of Special Chemical Sensation of Taste
Physiology of Special Chemical Sensation of TastePhysiology of Special Chemical Sensation of Taste
Physiology of Special Chemical Sensation of Taste
MedicoseAcademics
 
Vision-1.pptx, Eye structure, basics of optics
Vision-1.pptx, Eye structure, basics of opticsVision-1.pptx, Eye structure, basics of optics
Vision-1.pptx, Eye structure, basics of optics
Sai Sailesh Kumar Goothy
 
How STIs Influence the Development of Pelvic Inflammatory Disease.pptx
How STIs Influence the Development of Pelvic Inflammatory Disease.pptxHow STIs Influence the Development of Pelvic Inflammatory Disease.pptx
How STIs Influence the Development of Pelvic Inflammatory Disease.pptx
FFragrant
 
basicmodesofventilation2022-220313203758.pdf
basicmodesofventilation2022-220313203758.pdfbasicmodesofventilation2022-220313203758.pdf
basicmodesofventilation2022-220313203758.pdf
aljamhori teaching hospital
 
Aortic Association CBL Pilot April 19 – 20 Bern
Aortic Association CBL Pilot April 19 – 20 BernAortic Association CBL Pilot April 19 – 20 Bern
Aortic Association CBL Pilot April 19 – 20 Bern
suvadeepdas911
 
Cervical & Brachial Plexus By Dr. RIG.pptx
Cervical & Brachial Plexus By Dr. RIG.pptxCervical & Brachial Plexus By Dr. RIG.pptx
Cervical & Brachial Plexus By Dr. RIG.pptx
Dr. Rabia Inam Gandapore
 
Non-respiratory Functions of the Lungs.pdf
Non-respiratory Functions of the Lungs.pdfNon-respiratory Functions of the Lungs.pdf
Non-respiratory Functions of the Lungs.pdf
MedicoseAcademics
 
Role of Mukta Pishti in the Management of Hyperthyroidism
Role of Mukta Pishti in the Management of HyperthyroidismRole of Mukta Pishti in the Management of Hyperthyroidism
Role of Mukta Pishti in the Management of Hyperthyroidism
Dr. Jyothirmai Paindla
 
Netter's Atlas of Human Anatomy 7.ed.pdf
Netter's Atlas of Human Anatomy 7.ed.pdfNetter's Atlas of Human Anatomy 7.ed.pdf
Netter's Atlas of Human Anatomy 7.ed.pdf
BrissaOrtiz3
 

Recently uploaded (20)

Best Ayurvedic medicine for Gas and Indigestion
Best Ayurvedic medicine for Gas and IndigestionBest Ayurvedic medicine for Gas and Indigestion
Best Ayurvedic medicine for Gas and Indigestion
 
263778731218 Abortion Clinic /Pills In Harare ,
263778731218 Abortion Clinic /Pills In Harare ,263778731218 Abortion Clinic /Pills In Harare ,
263778731218 Abortion Clinic /Pills In Harare ,
 
micro teaching on communication m.sc nursing.pdf
micro teaching on communication m.sc nursing.pdfmicro teaching on communication m.sc nursing.pdf
micro teaching on communication m.sc nursing.pdf
 
Effective-Soaps-for-Fungal-Skin-Infections.pptx
Effective-Soaps-for-Fungal-Skin-Infections.pptxEffective-Soaps-for-Fungal-Skin-Infections.pptx
Effective-Soaps-for-Fungal-Skin-Infections.pptx
 
Colonic and anorectal physiology with surgical implications
Colonic and anorectal physiology with surgical implicationsColonic and anorectal physiology with surgical implications
Colonic and anorectal physiology with surgical implications
 
Ophthalmology Clinical Tests for OSCE exam
Ophthalmology Clinical Tests for OSCE examOphthalmology Clinical Tests for OSCE exam
Ophthalmology Clinical Tests for OSCE exam
 
Ozempic: Preoperative Management of Patients on GLP-1 Receptor Agonists
Ozempic: Preoperative Management of Patients on GLP-1 Receptor Agonists  Ozempic: Preoperative Management of Patients on GLP-1 Receptor Agonists
Ozempic: Preoperative Management of Patients on GLP-1 Receptor Agonists
 
A Classical Text Review on Basavarajeeyam
A Classical Text Review on BasavarajeeyamA Classical Text Review on Basavarajeeyam
A Classical Text Review on Basavarajeeyam
 
SURGICAL ANATOMY OF THE RETROPERITONEUM, ADRENALS, KIDNEYS AND URETERS.pptx
SURGICAL ANATOMY OF THE RETROPERITONEUM, ADRENALS, KIDNEYS AND URETERS.pptxSURGICAL ANATOMY OF THE RETROPERITONEUM, ADRENALS, KIDNEYS AND URETERS.pptx
SURGICAL ANATOMY OF THE RETROPERITONEUM, ADRENALS, KIDNEYS AND URETERS.pptx
 
Dehradun #ℂall #gIRLS Oyo Hotel 8107221448 #ℂall #gIRL in Dehradun
Dehradun #ℂall #gIRLS Oyo Hotel 8107221448 #ℂall #gIRL in DehradunDehradun #ℂall #gIRLS Oyo Hotel 8107221448 #ℂall #gIRL in Dehradun
Dehradun #ℂall #gIRLS Oyo Hotel 8107221448 #ℂall #gIRL in Dehradun
 
Thyroid Gland- Gross Anatomy by Dr. Rabia Inam Gandapore.pptx
Thyroid Gland- Gross Anatomy by Dr. Rabia Inam Gandapore.pptxThyroid Gland- Gross Anatomy by Dr. Rabia Inam Gandapore.pptx
Thyroid Gland- Gross Anatomy by Dr. Rabia Inam Gandapore.pptx
 
Physiology of Special Chemical Sensation of Taste
Physiology of Special Chemical Sensation of TastePhysiology of Special Chemical Sensation of Taste
Physiology of Special Chemical Sensation of Taste
 
Vision-1.pptx, Eye structure, basics of optics
Vision-1.pptx, Eye structure, basics of opticsVision-1.pptx, Eye structure, basics of optics
Vision-1.pptx, Eye structure, basics of optics
 
How STIs Influence the Development of Pelvic Inflammatory Disease.pptx
How STIs Influence the Development of Pelvic Inflammatory Disease.pptxHow STIs Influence the Development of Pelvic Inflammatory Disease.pptx
How STIs Influence the Development of Pelvic Inflammatory Disease.pptx
 
basicmodesofventilation2022-220313203758.pdf
basicmodesofventilation2022-220313203758.pdfbasicmodesofventilation2022-220313203758.pdf
basicmodesofventilation2022-220313203758.pdf
 
Aortic Association CBL Pilot April 19 – 20 Bern
Aortic Association CBL Pilot April 19 – 20 BernAortic Association CBL Pilot April 19 – 20 Bern
Aortic Association CBL Pilot April 19 – 20 Bern
 
Cervical & Brachial Plexus By Dr. RIG.pptx
Cervical & Brachial Plexus By Dr. RIG.pptxCervical & Brachial Plexus By Dr. RIG.pptx
Cervical & Brachial Plexus By Dr. RIG.pptx
 
Non-respiratory Functions of the Lungs.pdf
Non-respiratory Functions of the Lungs.pdfNon-respiratory Functions of the Lungs.pdf
Non-respiratory Functions of the Lungs.pdf
 
Role of Mukta Pishti in the Management of Hyperthyroidism
Role of Mukta Pishti in the Management of HyperthyroidismRole of Mukta Pishti in the Management of Hyperthyroidism
Role of Mukta Pishti in the Management of Hyperthyroidism
 
Netter's Atlas of Human Anatomy 7.ed.pdf
Netter's Atlas of Human Anatomy 7.ed.pdfNetter's Atlas of Human Anatomy 7.ed.pdf
Netter's Atlas of Human Anatomy 7.ed.pdf
 

Clinical Pharmacology of Drugs Used to Affect Renal Function

  • 1. Photo: Scanning electron micrograph of the glomerulus in a human kidney. From: Widmaier EP. Vander’s Human Physiology: The Mechanisms Of Body Function, 13th Ed. New York, NY: McGraw-Hill Companies, Inc., 2014: 490
  • 2. Marc Imhotep Cray, M.D. Learning Objectives: 1. List major types of diuretics and relate them to their sites of action. 2. List the major applications, toxicities, and the efficacy of thiazides, loop diuretics and potassium-sparing diuretics. 3. Describe two drugs that reduce potassium loss during diuresis. 4. Describe a therapy that will reduce calcium excretion in patients who have recurrent urinary stones. 5. Discuss the principle of force diuresis. 6. Describe drugs for reducing urine volume in nephrogenic diabetes insipidus. 7. Understand the usefulness of altering urine pH by drugs. 8. Discuss the mechanisms by which drugs and chemicals damage the kidney. 9. Understand how to select and prescribe drugs for patients with renal impairment. 2 Companion: Renal Pharmacology eNotes
  • 3. Marc Imhotep Cray, M.D. Some Relevant Drugs: 3 A. Carbonic Anhydrase Inhibitors Acetazolamide dichlorphenamide methazolamide dorzolamide B. Osmotic Diuretics mannitol C. Loop Diuretics furosemide bumetanide torsemide ethacrynic acid D. Thiazides & Thiazides-like chlorthalidone chlorothiazide hydrochlorothiazide metolazone indapamide E. Potassium-sparing diuretics spironolactone eplerenone triamterene amiloride F. ADH antagonists demeclocycline lithium lixivaptan tolvaptan conivaptan
  • 4. Marc Imhotep Cray, M.D. Topical Outline: 4 o Role of Renal System  Volume Homeostasis o General Principles of Diuretic Action o Individual Agents/Classes  High-efficacy (loop) diuretics  Moderate-efficacy diuretics  Low-efficacy diuretics  Osmotic diuretics  Carbonic Anhydrase Inhibitors o Adverse effects of diuretics and Drug-Drug Interactions o Alteration of Urine pH o Alkalinization o Acidification o Drugs and the Kidney and Prescribing in Renal Disease o ADH Antagonists  Clinical Cases and Discussions  Practice MCQs
  • 5. Marc Imhotep Cray, M.D. Key Abbreviations 5  PCT, Proximal convoluted tubule  DCT, Distal convoluted tubule  TAL, thick ascending limb of the loop of Henle  CCD, cortical collecting duct (including late DCT forming initial collecting duct)  MCD, medullary collecting duct  GFR, glomerular filtration rate  ENaC, epithelial sodium channel  NCC, Na-Cl cotransporter (formerly NCCT or thiazide-sensitive Na–Cl co-transporter)  NKCC2, Na–K–2Cl co-transporter  ROMK, rectifying outer medullary potassium channel
  • 6. Marc Imhotep Cray, M.D. High-Yield Terms to Learn 6  Bicarbonate diuretic A diuretic that selectively increases sodium bicarbonate excretion. Example: a carbonic anhydrase inhibitor  Diluting segment A segment of nephron that removes solute without water; TAL and DCT are active salt-reabsorbing segments that are not permeable by water  Hyperchloremic metabolic acidosis A shift in body electrolyte and pH balance involving elevated serum chloride, diminished bicarbonate concentration, and a decrease in pH in the blood. Typical result of bicarbonate diuresis  Hypokalemic metabolic alkalosis A shift in body electrolyte balance and pH involving a decrease in serum potassium and an increase in blood pH. Typical result of loop and thiazide diuretic actions
  • 7. Marc Imhotep Cray, M.D. High-Yield Terms to Learn cont. 7  Nephrogenic diabetes insipidus Loss of urine-concentrating ability in kidney caused by lack of responsiveness to ADH (ADH is normal or high)  Pituitary diabetes insipidus Loss of urine-concentrating ability in kidney caused by lack of ADH (ADH is low or absent)  Potassium-sparing diuretic A diuretic that reduces exchange of potassium for sodium in collecting tubule; a drug that increases sodium and reduces potassium excretion. Example: aldosterone antagonists  Uricosuric diuretic A diuretic that increases uric acid excretion, , usually by inhibiting uric acid reabsorption in the proximal tubule. Example: ethacrynic acid
  • 8. Marc Imhotep Cray, M.D. Key Concepts in Clinical Renal Pharmacology 8  Diuretic drugs: their sites and modes of action, classification, adverse effects and uses in cardiac, hepatic, renal and other conditions.  Carbonic anhydrase inhibitors.  Cation-exchange resins and their uses.  Alteration of urine pH.  Drugs and the kidney.  Adverse effects.  Drug-induced renal disease: by direct and indirect biochemical effects and by immunological effects.  Prescribing for renal disease: adjusting the dose according to the characteristics of the drug and to the degree of renal impairment.  Nephrolithiasis and its management.  Pharmacological aspects of micturition.  Benign prostatic hyperplasia.  Erectile dysfunction.
  • 9. Marc Imhotep Cray, M.D. Role of Renal System 9  The kidneys comprise only 0.5% of body-weight, yet they receive 25% of the cardiac output.  Drugs that affect renal function have important roles in cardiac failure and hypertension  Disease of kidney must be taken into account when prescribing drugs that are eliminated by it  drugs can damage kidney and disease of kidney affects responses to drugs (will be covered elsewhere)
  • 10. Marc Imhotep Cray, M.D. Role of Renal System (2): Volume Homeostasis 10 Kidneys are part of an integrated homeostatic mechanism for maintaining volume of extracellular fluid (ECF) and thus mean arterial pressure (MAP) Other organs involved in this mechanism include:  Heart (eg, cardiac output and heart rate),  CNS (eg, sympathetic tone and ADH release),  Lungs (eg, conversion of angiotensin I to angiotensin II), and  Adrenal gland (eg, release of aldosterone)
  • 11. Marc Imhotep Cray, M.D. Volume Homeostasis (2) 11 Several feedback control mechanisms operate among components of this control mechanism ensure responses to  volume expansion (increased extracellular fluid) and  volume contraction (decreased extracellular fluid) Design of drugs that selectively target components of this system has led to major advances in therapy for cardiovascular diseases such as hypertension and heart failure  Discussed in Unit 4 Drugs Used In Disorders of the Cardiovascular System
  • 12. Marc Imhotep Cray, M.D. 12 Volume expansion feedback control Raff RB, Rawls SM, Beyzarov EP. Netter's Illustrated Pharmacology, Updated Edition. Saunders, 2014
  • 13. Marc Imhotep Cray, M.D. 13 Volume contraction feedback control Raff RB, Rawls SM, Beyzarov EP. Netter's Illustrated Pharmacology, Updated Edition. Saunders, 2014
  • 14. Marc Imhotep Cray, M.D. General Principles of Diuretic Action 14  Definition: A diuretic is any substance that increases urine and solute excretion  This wide definition includes substances not commonly thought of as diuretics, e.g. water  To be therapeutically useful a diuretic should  increase output of sodium as well as of water because diuretics are normally required to remove edema fluid, composed of water and solutes (of which sodium is most important)
  • 15. Marc Imhotep Cray, M.D. GP of Diuretic Action (2) 15 Each day body produces 180 L of glomerular filtrate which is modified in its passage down renal tubules to appear as 1.5 L of urine  Thus, if reabsorption of tubular fluid falls by 1%, urine output doubles  Most clinically useful diuretics are organic anions transported directly from blood into tubular fluid  Following is a brief account of tubular function with particular reference to sodium transport  Intended to help to explain where and how diuretic drugs act o it should be understood with reference to Figure following text
  • 16. Marc Imhotep Cray, M.D. GP of Diuretic Action (3) 16 Sites and modes of action Proximal convoluted tubule (PCT)  Some 65% of filtered sodium is actively transported from lumen of PCT by sodium pump (Na+, K+-ATPase)  Chloride is absorbed passively, accompanying sodium  Bicarbonate is also absorbed through an action involving carbonic anhydrase  These solute shifts give rise to iso-osmotic reabsorption of water with result that more than 70% of glomerular filtrate is returned to blood from this section of nephron  Epithelium of PCT is described as “leaky” because of its free permeability to water and a number of solutes
  • 17. Marc Imhotep Cray, M.D. GP of Diuretic Action (4) 17 Sites and modes of action Proximal convoluted tubule cont.  Osmotic diuretics such as mannitol are non-resorbable solutes which retain water in tubular fluid (Site 1 in Figure)  Their effect is to increase water rather than sodium loss reflected in their special use acutely to reduce intracranial or intraocular pressure and not states associated with sodium overload
  • 18. Marc Imhotep Cray, M.D. GP of Diuretic Action (5) 18  Tubular fluid now passes into loop of Henle where 25% of filtered sodium is reabsorbed  There are two populations of nephron:  those with short loops confined to cortex, and  juxtamedullary nephrons whose long loops penetrate deep into medulla  are concerned principally with water conservation o following discussion refers to these long loops Sites and modes of action Loop of Henle
  • 19. Marc Imhotep Cray, M.D. GP of Diuretic Action (6) 19  Physiologic changes best understood by first considering ascending limb  In TAL (Site 2, slide 25) sodium and chloride ions are transported from tubular fluid into interstitial fluid by the three-ion co-transporter system (i.e. Na+/K+/2Cl- called NKCC2) driven by sodium pump o dependent on potassium returning to lumen through rectifying outer medullary potassium (ROMK) channel otherwise K+ would be rate limiting  As tubule epithelium is “tight” here, i.e. impermeable to water tubular fluid becomes dilute interstitium becomes hypertonic and  fluid in adjacent descending limb, which is permeable to water becomes more concentrated as it approaches tip of loop o b/c hypertonic interstitial fluid sucks water out of this limb of tubule Sites and modes of action Loop of Henle cont.
  • 20. Marc Imhotep Cray, M.D. 20 GP of Diuretic Action (7) Sites and modes of action, Loop of Henle cont.  High osmotic pressure in medullary interstitium is sustained by descending and ascending vasa recta (long blood vessels of capillary thickness that lie close to loops of Henle and act as countercurrent exchangers) for incoming bld receives sodium from outgoing bld  Furosemide, bumetanide, torasemide and ethacrynic acid act principally at TAL (Site 2, slide 25) by inhibiting the three-ion transporter (NKCC2) thus preventing sodium ion reabsorption and lowering osmotic gradient betw. cortex and medulla results in formation of large volumes of dilute urine  Hence, called “loop” diuretics
  • 21. Marc Imhotep Cray, M.D. 21 GP of Diuretic Action (8) Sites and modes of action Distal convoluted tubule (DCT)  Ascending limb of the loop then re-enters renal cortex where its morphology changes into thin-walled DCT (Site 3, slide 25)  Here uptake is still driven by sodium pump but sodium and chloride are taken up through a different transporter Na-Cl cotransporter, called NCC (formerly NCCT)  Both ions are rapidly removed from interstitium b/c cortical blood flow is high and there are no vasa recta present  Epithelium is also tight at Site 3 and consequently urine becomes more dilute  Thiazides act at this region of cortical diluting segment by blocking NCC transporter
  • 22. Marc Imhotep Cray, M.D. 22  In collecting duct (Site 4, slide 25), Na ions are exchanged for K and H ions  Na ions enter through epithelial Na channel (called ENaC), which is stimulated by aldosterone  The aldosterone (mineralocorticoid) receptor is inhibited by competitive receptor antagonist spironolactone whereas  sodium channel is inhibited by amiloride and triamterene  All three of these diuretics are potassium sparing b/c K+ is normally secreted through K+ channel, ROMK (see Figure), down potential gradient created by sodium reabsorption GP of Diuretic Action (9) Sites and modes of action Cortical collecting duct (CCD)
  • 23. Marc Imhotep Cray, M.D. 23  All other diuretics, acting proximal to Site 4, cause potassium Loss b/c they dump sodium into collecting duct  Removal of this sodium through ENaC increases potential gradient for potassium secretion through ROMK  K+ sparing diuretics are weak diuretics b/c Site 4 is normally responsible for “only” 2–3% of sodium reabsorption  cause less sodium loss than thiazides or loop diuretics  NB: Although ENaC does not have capacity to compensate for lg. Na losses (e.g. loop diuretic usage) it is main site of physiologic control (via aldosterone) over sodium loss GP of Diuretic Action (10) Sites and modes of action Cortical collecting duct (CCD) cont.
  • 24. Marc Imhotep Cray, M.D. 24  Collecting ducts then travels back through medulla to reach papilla in doing so it passes through a gradient of increasing osmotic pressure which draws water out of tubular fluid  This final conc. of urine is under influence of ADH = increases water permeability by increasing expression of specific water channels (or aquaporins) o In ADH’s absence water remains in collecting duct  Ethanol causes diuresis by inhibiting release of ADH from posterior pituitary GP of Diuretic Action (11) Sites and modes of action Cortical collecting duct (CCD) cont. NB: Diuresis may also be achieved by extrarenal mechanisms, by raising cardiac output and increasing renal blood flow, e.g. with dobutamine and dopamine.
  • 25. 25Bennett PN, Brown MJ and Sharma P. Clinical Pharmacology 11th Ed. Edinburgh: Churchill Livingstone, 2012. Sites of action of diuretic drugs Inset show transporters and ion channels targeted in tubular cells at these sites • ENaC, epithelial sodium channel; • NCCT, thiazide-sensitive Na–Cl co-transporter; • NKCC2, Na–K–2Cl co- transporter; • ROMK, rectifying outer medullary potassium channel
  • 26. Marc Imhotep Cray, M.D. GP of Diuretic Action (12) Classification 26  Maximum efficacy in removing salt and water that any diuretic achieves is dependent on its site of action, thus it is appropriate to rank diuretics according to their natriuretic capacity (as set out in slides that follow)  Classes: 1. High efficacy 2. Moderate efficacy 3. Low efficacy NB: Percentages refer to highest fractional excretion of filtered sodium under carefully controlled conditions and should not be taken to represent average fractional sodium loss during clinical use.
  • 27. Marc Imhotep Cray, M.D. GP of Diuretic Action (13) Classification 27 1. High efficacy Furosemide and other “loop” diuretics can cause up to 25% of filtered sodium to be excreted  Their action impairs powerful urine-concentrating mechanism of loop of Henle and confers higher efficacy compared with drugs that act in relatively hypotonic cortex Progressive increase in dose is matched by increasing diuresis,  i.e. they have a “high ceiling” of effect  they are so effective that over-treatment can readily dehydrate patient Loop diuretics remain effective at a glomerular filtration rate (GFR) below 10 mL/min (nml 120 mL/min)
  • 28. Marc Imhotep Cray, M.D. GP of Diuretic Action (14) Classification 28 2. Moderate efficacy  The thiazide family, including chlorthalidone, chlorothiazide, hydrochlorothiazide, metolazone and indapamide, cause 5- 10% of filtered sodium load to be excreted  Increasing dose produces relatively little added diuresis compared to loop diuretics  i.e. they have a “low ceiling” of effect  Cease to be effective once GFR has fallen below 20 mL/min (except metolazone)
  • 29. Marc Imhotep Cray, M.D. GP of Diuretic Action (15) Classification 29 3. Low efficacy Triamterene, amiloride and spironolactone cause 2–3% of filtered sodium to be excreted  They are potassium sparing and combine with more efficacious diuretics to prevent potassium loss, which other diuretics cause Osmotic diuretics, e.g. mannitol, also fall into this category
  • 30. Marc Imhotep Cray, M.D. GP of Diuretic Action (16) Indications 30  Edema states associated  with sodium overload, e.g. cardiac, renal or hepatic disease, and also  without sodium overload, e.g. acute pulmonary edema following myocardial infarction NB: Edema may also be localized, e.g.  angioedema over face and neck or around ankles with some calcium channel blockers, or  due to low plasma albumin, or immobility in elderly  in none of these circumstances is a diuretic indicated  Hypertension, by reducing intravascular volume and other mechanisms too, e.g. reduction of sensitivity to noradrenergic vasoconstriction
  • 31. Marc Imhotep Cray, M.D. 31  Hypercalcemia Furosemide reduces calcium reabsorption in ascending limb of loop of Henle action may be utilised in emergency reduction of raised plasma calcium levels, in addition to rehydration and other measures  Idiopathic hypercalciuria, a common cause of renal stone disease, may be reduced by thiazide diuretics  Syndrome of inappropriate secretion of antidiuretic hormone secretion (SIADH) may be treated with furosemide if there is a dangerous degree of volume overload  Nephrogenic diabetes insipidus, paradoxically, may respond to diuretics which, by contracting vascular volume, increase salt and water reabsorption in PCT thus reduce urine volume GP of Diuretic Action (17) Indications cont.
  • 32. Marc Imhotep Cray, M.D. Individual Agents/Classes 32 High-efficacy (loop) diuretics Furosemide… Moderate-efficacy diuretics Thiazides… Low-efficacy (K+ sparing) diuretics Spironolactone… Osmotic diuretics Mannitol… Carbonic Anhydrase Inhibitors Acetazolamide…
  • 33. Marc Imhotep Cray, M.D. High-efficacy (loop) diuretics Furosemide (Prototype) Furosemide acts on thick portion of ascending limb of the loop of Henle (Site 2 in slide 25) to produce effects described above  b/c more sodium is delivered to DCT & CD (Site 4 in slide 25), exchange with potassium leads to urinary potassium loss and hypokalemia  Magnesium and calcium loss are increased by furosemide to same extent as sodium effect on calcium is utilized in emergency management of hypercalcemia Pharmacokinetics  Absorption from GIT is subject to considerable intra- and inter- individual variation and it is highly bound to plasma proteins  t½ is 2 hrs rises to over 10 h in renal failure 33
  • 34. Marc Imhotep Cray, M.D. High-efficacy (loop) diuretics Furosemide cont. Uses  very successful for the relief of edema  Urine production rises progressively with increasing dose  Taken orally it acts within an hour and diuresis lasts up to 6 h Caution  Enormous urine volumes can result and over-treatment may lead to hypovolemia and circulatory collapse  Given intravenously it acts within 30 min and can relieve acute pulmonary Edema partly by a venodilator action which precedes diuresis  Important feature  retains efficacy even at a low GFR (10 mL/min or less) 34
  • 35. Marc Imhotep Cray, M.D. High-efficacy (loop) diuretics Furosemide cont. Adverse effects uncommon, apart from excess of therapeutic effect (electrolyte disturbance and hypotension due to low plasma volume) and  Nausea  Pancreatitis and,  rarely, deafness, which is usually transient and associated with rapid IV injection in renal failure Non-steroidal anti-inflammatory drugs (NSAIDs), notably indomethacin, reduce furosemide-induced diuresis by inhibiting formation of vasodilator prostaglandins in kidney 35
  • 36. Marc Imhotep Cray, M.D. High-efficacy (loop) diuretics cont. Bumetanide, piretanide and ethacrynic acid are similar to furosemide Bumetanide may be preferred over furosemide in heart failure b/c of more predictable oral absorption Ethacrynic acid is less widely used as it is more prone to cause adverse effects, especially nausea and deafness (ototoxicity)  Not sulfa containing as are other loop diuretics, thus useful option in sulfa-allergic pts. Torasemide is an effective antihypertensive agent at lower (non-natriuretic) doses (2.5–5 mg/day) than those used for edema (5–40 mg/day) 36
  • 37. Marc Imhotep Cray, M.D. NB Sidebar: Sulfa drugs and sulfa allergies  Sulfa containing drugs: Sulfonamides antibiotics, Sulfasalazine, Probenecid, Furosemide, Acetazolamide, Celecoxib, Thiazides, Sulfonylureas (Scary Sulfa Pharm FACTS)  Clinical manifestations of sulfa allergies= Pts. w sulfa allergies may develop:  Fever  Urinary tract infection  Stevens-Johnson syndrome (SJS)  Hemolytic anemia, thrombocytopenia agranulocytosis, and  Urticaria (hives)  Symptoms range from mild to life threatening
  • 38. Marc Imhotep Cray, M.D. Moderate-efficacy diuretics Thiazides Thiazides depress salt reabsorption in DCT (Site 3 in slide 25), i.e. upstream of region of sodium–potassium exchange at CD (Site 4 in slide 25)  Hence , have important effect of raising potassium excretion  Thiazides lower blood pressure, initially due to a reduction in intravascular volume but chronically by a reduction in peripheral vascular resistance  accompanied by diminished responsiveness of vascular smooth muscle to Epi/NE  also have a direct action on vascular smooth muscle membranes 37
  • 39. Marc Imhotep Cray, M.D. Moderate-efficacy diuretics Thiazides cont. Uses  given for mild cardiac failure and mild hypertension, or for more severe degrees of HTN, in combination with other drugs Pharmacokinetics  Thiazides are well absorbed orally and most begin to act within an hour  Differences among numerous derivatives lie in duration of action  Relatively water-soluble agents, e.g. chlorothiazide, hydrochlorothiazide (HCTZ), are most rapidly eliminated, peak effect within 4–6 h and passing off by 10–12 h o excreted unchanged in urine and active secretion by PCT contributes to high renal clearance and t½ of less than 4 h 38
  • 40. Marc Imhotep Cray, M.D. Moderate-efficacy diuretics Thiazides cont. Pharmacokinetics  Relatively lipid-soluble members, e.g. polythiazide, hydroflumethiazide, distribute more widely into body tissues and act for >24 h o can be problematic if used for diuresis, but no evidence this property makes them more effective at controlling hypertension With exception of metolazone, thiazides are not effective when renal function is impaired (GFR <20 mL/min), b/c they are not filtered in sufficient conc. to inhibit NCC (Na-Cl cotransporter) 39
  • 41. Marc Imhotep Cray, M.D. Moderate-efficacy diuretics Thiazides cont. Adverse effects (Adverse effects in general discussed in a section to follow)  Rashes (sometimes photosensitive)  thrombocytopenia and  agranulocytosis occur  Thiazide-type drugs can increase total plasma cholesterol concentration  But in long-term use this is less than 5%, even at high doses  Questions about appropriateness of thiazides for mild hypertension, of which ischemic heart disease is a common complication, are laid to rest by their proven success in randomized outcome comparisons 40
  • 42. Marc Imhotep Cray, M.D. Moderate-efficacy diuretics Diuretics related to thiazides Several compounds, not strictly thiazides, share structural similarities and act at same site on nephron Overall, these agents have a longer duration of action, are used for edema and hypertension, and their profile of adverse effects similar to thiazides  Chlorthalidone acts for 48–72 h after a single oral dose  Indapamide is structurally related to chlortalidone but lowers blood pressure at subdiuretic doses perhaps by altering calcium flux in vascular smooth muscle  Metolazone is effective when renal function is impaired o It potentiates diuresis produced by furosemide and combination can be effective in resistant edema although risk of hypokalemia is very high 41
  • 43. Marc Imhotep Cray, M.D. Low-efficacy diuretics  Spironolactone (Aldactone) is structurally similar to aldosterone and competitively inhibits its action in distal tubule (exchange of potassium for sodium, Site 4 of slide 25)  Excessive secretion of aldosterone contributes to fluid retention in  hepatic cirrhosis  nephrotic syndrome  congestive heart failure and  primary hypersecretion (Conn’s syndrome)  Spironolactone is also useful in treatment of resistant hypertension  increased aldosterone sensitivity is increasingly recognized as a contributory factor 42
  • 44. Marc Imhotep Cray, M.D. Low-efficacy diuretics cont.  Spironolactone has a short t½ (1.6 h), being extensively metabolized, and its prolonged diuretic effect is due to most significant active metabolite, canrenone (t½ 17 h)  relatively ineffective when used alone  more efficient when combined w a drug that reduces sodium  given orally in one or more doses totaling 100–200 mg/day  Maximum diuresis may not occur for up to 4 days  Spironolactone (and amiloride and triamterene) usefully reduces K+ loss caused by loop diuretics Warnings:  combination with another K+ sparing diuretic must be avoided as hyperkalemia will result  Dangerous K+ retention is particularly likely if spironolactone is given to pts. with impaired renal function 43
  • 45. Marc Imhotep Cray, M.D. Low-efficacy diuretics cont. Adverse effects  Estrogenic effects are major limitation to its long-term use  Randomized Aldactone Evaluation Study (RALES) even 25 mg/day caused breast tenderness or enlargement in 10% of men (N Engl J Med. 1999 Sep 2;341(10):709-17)  Women may also report breast discomfort or menstrual irregularities, including amenorrhea  Minor gastrointestinal upset and increased risk of gastroduodenal ulcer and bleeding  Usually reversible on stopping drug  Spironolactone is reported to be carcinogenic in rodents, but clinical experience suggest tit is safe in humans  Nevertheless, UK license for its use in essential hypertension was withdrawn (i.e. possible use long term in a pt. group including relatively young), but is retained for other indications 44
  • 46. Marc Imhotep Cray, M.D. Low-efficacy diuretics cont.  Eplerenone is a spironolactone analog licensed for use in heart failure  free of estrogenic effects; b/c of its lower affinity for estrogen receptor  It is useful in pts who need an aldosterone-receptor blocking agent, but are intolerant of endocrine effects of spironolactone 45
  • 47. Marc Imhotep Cray, M.D. Low-efficacy diuretics cont. Amiloride blocks ENaC sodium channels in distal tubule  Action complements thiazides with which it is frequently combined to increase sodium loss and limit potassium loss  Example, coamilozide (amiloride 2.5–5 mg plus hydrochlorothiazide 25–50 mg) is used for hypertension or edema maximum effect of amiloride occurs about 6 h after an oral dose, with a duration of action greater than 24 h (t½ 21 h) oral dose is 5–20 mg daily 46
  • 48. Marc Imhotep Cray, M.D. Low-efficacy diuretics cont.  Triamterene (Dytac) is a potassium-sparing diuretic with an action and use similar to amiloride (blocks ENaC sodium channels in DCT)  Diuretic effect extends over 10 h Adverse effects  Gastrointestinal upsets occur Drug-drug interaction  Reversible, non-oliguric renal failure may occur when triamterene is used with indomethacin (and other NSAIDs)  may also give urine a blue coloration 47
  • 49. Marc Imhotep Cray, M.D. Osmotic diuretics  Osmotic diuretics are small molecular weight substances that are filtered by the glomerulus but not reabsorbed by renal tubule and thus increase osmolarity of tubular fluid  Thus they prevent reabsorption of water (and also, by more complex mechanisms, of sodium) principally in PCT and also loop of Henle  Result is urine volume increases according to load of osmotic diuretic 48
  • 50. Marc Imhotep Cray, M.D. Osmotic diuretics cont.  Mannitol, a polyhydric alcohol (mol. wt. 452), is used most commonly given intravenously  In addition to effect on kidney, mannitol encourages movement of water from inside cells to extracellular fluid ECF (including circulatory volume) thus transiently expanded before diuresis occurs Uses: Rapid reduction of intracranial or intraocular pressure, and to maintain urine flow to prevent renal tubular necrosis Contraindications:  b/c mannitol increases circulatory volume, it is contraindicated in congestive cardiac failure (CHF) and pulmonary edema (PE) 49
  • 51. Marc Imhotep Cray, M.D. Carbonic Anhydrase Inhibitors  The enzyme carbonic anhydrase facilitates reaction betw. CO2 and H2O to form carbonic acid (H2CO3), which then breaks down to hydrogen (H+) and bicarbonate (HCO3-) ions  This process is fundamental to production of either acid or alkaline secretions  high concentrations of CA are present in gastric mucosa, pancreas, eye and kidney  MOA b/c number of H+ ions available to exchange with Na+ in PCT is reduced, sodium loss and diuresis occur  But HCO3- reabsorption from tubule is also reduced, and its loss in urine leads within days to metabolic acidosis attenuates diuretic response to carbonic anhydrase inhibition o Consequently, inhibitors of CA are obsolete as diuretics • Still have specific uses  Acetazolamide is most widely used CAI 50
  • 52. Marc Imhotep Cray, M.D. Carbonic Anhydrase Inhibitors cont. Reduction of intraocular pressure  action is not due to diuresis  rather, formation of aqueous humor is an active process requiring a supply of bicarbonate ions which depends on carbonic anhydrase  Inhibition of CA reduces formation of aqueous humor and lowers IOP o this is a local action and is not affected by development of acid–base changes elsewhere in body, i.e. tolerance does not develop  In pts. w acute glaucoma, acetazolamide taken either PO or IV  Acetazolamide is not recommended for long-term use b/c of risk of hypokalemia and acidosis but brinzolamide or dorzolamide are effective as eye drops, well tolerated, and thus suitable for chronic use in glaucoma 51
  • 53. Marc Imhotep Cray, M.D. Carbonic Anhydrase Inhibitors cont. Acetazolamide for High-altitude (mountain) sickness  High-altitude (mountain) sickness may affect unacclimatized people at altitudes over 3000 meters, especially after rapid ascent  symptoms range from  nausea  lassitude and headache to  pulmonary and cerebral edema  Initiating cause is hypoxia:  at high altitude, normal hyperventilatory response to falling oxygen tension is inhibited b/c alkalosis is also induced  Acetazolamide induces metabolic acidosis increases respiratory drive, notably at night when apnetic attacks may occur, and thus helps to maintain arterial oxygen tension 52
  • 54. Marc Imhotep Cray, M.D. CAIs cont., acetazolamide for high-altitude Dosage  Usual dose is 125–250 mg twice daily, given orally on day before ascent and continued for 2 days after reaching intended altitude  250 mg twice daily is used to treat established high-altitude sickness, combined with a return to a lower altitude (Note: this is an unlicensed indication in UK)  As an alternative or in addition to acetazolamide dexamethasone may be used:  2 mg q6 hrs. for prevention, and  4 mg q6 hrs. for treatment 53
  • 55. Marc Imhotep Cray, M.D. CAIs cont., acetazolamide Acetazolamide has two other uses 1. In periodic paralysis, where sudden falls in plasma K+ conc. occur due to its exchange with Na+ in cells  rise in plasma H+ caused by acetazolamide provides an alternative cation to K+ for exchange with Na+ 2. Acetazolamide may be used occasionally as a second-line drug for tonic–clonic and partial epileptic seizures 54
  • 56. Marc Imhotep Cray, M.D. CAIs cont., acetazolamide Adverse effects  High doses of acetazolamide may cause  drowsiness and fever  rashes (it is a sulfonamide-type drug) and  paranesthesia may occur (from the acidosis)  blood disorders have been reported  Renal calculi may develop, b/c urine calcium is in less soluble form, owing to low citrate content of urine a consequence of metabolic acidosis  Dichlorphenamide is a similar, but a more potent, inhibitor of carbonic anhydrase 55
  • 57. Marc Imhotep Cray, M.D. Adverse effects of diuretics and Drug-Drug Interactions 57
  • 58. Marc Imhotep Cray, M.D. Potassium depletion 57 Diuretics that act at Sites 1, 2 and 3 of slide 25 cause more sodium to reach sodium–potassium exchange site in distal tubule (Site 4) and so increase potassium excretion This subject warrants discussion b/c hypokalemia may cause cardiac arrhythmia in patients at risk (e.g. receiving digoxin) Safe lower limit for plasma potassium conc. is 3.5 mEq/L Whether or not diuretic therapy causes significant lowering of serum potassium levels depends both on drug and on circumstances in which it is used following slides explain more
  • 59. Marc Imhotep Cray, M.D. Potassium depletion 58  The loop diuretics produce a smaller fall in serum K+ conc. than do thiazides, for equivalent diuretic effect, but have a greater capacity for diuresis, i.e. higher efficacy especially in large dose so are associated with greater decline in potassium levels  If diuresis is brisk and continuous, clinically important potassium depletion is likely to occur  NB: Hypokalemia predisposes pts Tx with cardiac glycosides (digoxin) to toxicity (permissive for digoxin binding at K+ binding site on Na+/K+ ATPase)
  • 60. Marc Imhotep Cray, M.D. Potassium depletion cont. 59  Low dietary intake of potassium predisposes to hypokalemia risk is particularly notable in elderly, many of whom ingest less than 50 mEq per day (dietary normal is 80 mEq).  Hypokalemia may be aggravated by other drugs, e.g. β2- agonists, theophylline, corticosteroids, amphotericin  Hypokalemia during diuretic therapy is also more likely in hyperaldosteronism  whether primary or more commonly secondary to severe liver disease, congestive heart failure or nephrotic syndrome
  • 61. Marc Imhotep Cray, M.D. Potassium depletion cont.  Potassium loss occurs with diarrhea, vomiting and small bowel fistula and may be aggravated by diuretic therapy  When a thiazide diuretic is used for hypertension no case for routine prescription of a potassium supplement if no predisposing factors are present 60
  • 62. Marc Imhotep Cray, M.D. Potassium depletion cont. Potassium depletion can be minimized or corrected by:  Maintaining a good dietary potassium intake (fruits, fruit juices, vegetables)  Combining a potassium-depleting with a potassium sparing agent  Intermittent use of potassium-losing drugs, i.e. drug holidays  Potassium supplements: KCl preferred b/c chloride is principal anion excreted along with sodium when high-efficacy diuretics are used Potassium-sparing diuretics defend plasma potassium more effectively than potassium supplements NB: All forms of K are irritant to GIT, and in esophagus may cause ulceration. Elderly, in particular, should be warned never to take such tablets dry but always with a large cupful of liquid and sitting upright or standing. 61
  • 63. Marc Imhotep Cray, M.D. Hyperkalemia Hyperkalemia may occur, esp. if a K+ sparing diuretic is given to a patient with impaired renal function  ACE inhibitors and ARBs can cause a increase in plasma K+ levels  They may cause dangerous hyperkalemia if combined with KCl supplements or other potassium sparing drugs, in presence of impaired renal function o However, w suitable monitoring combination can be used safely, as was well illustrated by RALES trial  Cyclosporine, tacrolimus, indomethacin and possibly other NSAIDs may cause hyperkalemia w potassium-sparing diuretics 62
  • 64. Marc Imhotep Cray, M.D. Treatment of hyperkalemia Tx of ↑K+ depends on severity and following measures are appropriate:  Any potassium-sparing diuretic should be discontinued  A cation-exchange resin, e.g. Polystyrene sulfonate resin can be used orally (more effective than rectally), to remove body potassium by gut  K+ may be moved rapidly from plasma into cells by giving: o sodium bicarbonate, 50 mL 8.4% solution through a central line, and repeated in a few minutes if characteristic ECG changes persist o glucose, 50 mL 50% solution, plus 10 units regular insulin by i.v. infusion o nebulized β2-agonist, salbutamol 5–10 mg, is effective in stimulating pumping of potassium into skeletal muscle
  • 65. Marc Imhotep Cray, M.D. Tx of hyperkalemia cont.  In presence of ECG changes, calcium gluconate, 10 mL of 10% solution, given i.v. and repeated if necessary in a few minutes o it has no effect on serum potassium but opposes myocardial effect of a raised serum potassium level o Caution • Calcium may potentiate digoxin and should be used cautiously, if at all, in a patient taking this drug • NB: Sodium bicarbonate and calcium salt must not be mixed in a syringe or reservoir b/c calcium precipitates  Dialysis may be needed in refractory cases & is highly effective
  • 66. Marc Imhotep Cray, M.D. Hypovolemia  Hypovolemia can result from over-treatment  Acute loss of excessive fluid leads to postural hypotension and dizziness  A more insidious state of chronic hypovolemia can develop, especially in elderly  After initial benefit, pt. becomes sleepy and lethargic  Blood urea concentration (BUN) rises and Na+ conc. may be low o Renal failure may result 63
  • 67. Marc Imhotep Cray, M.D. Urinary retention Urinary retention  Sudden vigorous diuresis can cause acute retention of urine in presence of bladder neck obstruction  e.g. due to prostatic enlargement 64
  • 68. Marc Imhotep Cray, M.D. Hyponatremia  Hyponatremia may result if Na+ loss occurs in pts who drink a large quantity of water when taking a diuretic  Other mechanisms are involved, including enhancement of ADH release  Such pts. have reduced total body Na+ and ECF vol. and are edema free  Discontinuing diuretic and restricting water intake are effective  The condition should be distinguished from hyponatremia with edema, which develops in patients with CHF, cirrhosis or nephrotic syndrome  Here salt and water intake should be restricted b/c ECF volume is expanded  Combination of a potassium-sparing diuretic and ACE inhibitor can also cause severe hyponatremia  more commonly than life-threatening hyperkalemia 65
  • 69. Marc Imhotep Cray, M.D. Urate retention Urate retention with hyperuricemia and, sometimes, clinical gout occurs with thiazides and loop diuretics  Effect is unimportant or negligible with low-efficacy diuretics, e.g. amiloride and spironolactone  Two mechanisms responsible  First, diuretics cause volume depletion, reduction in glomerular filtration and increased absorption of almost all solutes in proximal tubule, including urate  Second, diuretics and uric acid are organic acids and compete for transport mechanism that pumps such substances from blood into tubular fluid  Diuretic-induced hyperuricemia can be prevented by allopurinol or probenecid (which also antagonizes diuretic efficacy by reducing their transport into urine) 66
  • 70. Marc Imhotep Cray, M.D. Magnesium deficiency Magnesium deficiency: Loop and thiazide diuretics cause significant urinary loss of magnesium  potassium-sparing diuretics cause magnesium retention Magnesium deficiency brought about by diuretics is rarely severe enough to induce classic picture of neuromuscular irritability and tetany  but cardiac arrhythmias, mainly of ventricular origin, do occur  respond to repletion of magnesium (2 g of Mg2+ is given as 4 mL 50% magnesium sulfate infused i.v. over 10–15 min followed by up to 70 mmol infused over the next 24 h) 67
  • 71. Marc Imhotep Cray, M.D. Carbohydrate intolerance Carbohydrate intolerance is caused by those diuretics that produce prolonged hypokalemia, i.e. loop and thiazide type Mechanism May affect depolarization and entry of calcium into islet cells which is necessary to stimulate formation and release of insulin so glucose intolerance is probably due to secondary insulin deficiency  Insulin requirements thus increase in established diabetics and disease may become manifest in latent diabetics  effect is generally reversible over several months 68
  • 72. Marc Imhotep Cray, M.D. Calcium homeostasis Renal calcium loss is increased by loop diuretics  In short term this is not a serious disadvantage and furosemide may be used in management of hypercalcemia after rehydration achieved  In long term hypocalcaemia may be harmful, especially in elderly patients, who tend in any case to be in negative calcium balance Thiazides, by contrast, decrease renal excretion of calcium  this property may influence choice of diuretic in a potentially calcium- deficient or osteoporotic individual as thiazide use is associated with a reduced risk of hip fracture in elderly  Hypocalciuric effect of thiazides has also been used effectively in patients with idiopathic hypercalciuria commonest metabolic cause of renal stones 69
  • 73. Marc Imhotep Cray, M.D. Drug-Drug Interactions  Loop diuretics (especially as intravenous boluses) potentiate ototoxicity of aminoglycosides and nephrotoxicity of some cephalosporins  NSAIDs tend to cause sodium retention, which counteracts the effect of diuretics mechanism may involve inhibition of renal prostaglandin formation  Diuretic treatment of a patient taking lithium can precipitate toxicity from this drug (increased sodium loss is accompanied by reduced lithium excretion)  Other drugs that may induce hyperkaliemia, hypokalemia, hyponatremia or glucose intolerance with diuretics are described above 70
  • 74. Marc Imhotep Cray, M.D. Alteration Of Urine pH Alteration of urine pH by drugs is sometimes desirable  most common reason is in treatment of poisoning (a fuller account is given in poisoning and overdose) A summary of main indications follows:  Alkalinization of urine:  increases elimination of salicylate, phenobarbital and chlorophenoxy herbicides  treats crystal nephropathy by increasing drug solubility, e.g. of methotrexate, sulfonamides and triamterene  reduces irritation of an inflamed urinary tract  discourages growth of certain organisms, e.g. Escherichia coli 71
  • 75. Marc Imhotep Cray, M.D. Alteration Of Urine pH cont.  Urine can be made alkaline by sodium bicarbonate i.v., or by potassium citrate by mouth Caution: Sodium overload may exacerbate cardiac failure, and sodium or potassium excess are dangerous when renal function is impaired Acidification of urine:  used as a test for renal tubular acidosis  increases elimination of amphetamine, MDMA or “Ecstasy”, quinine and phencyclidine (very rarely needed)  Oral NH4Cl, taken w food to avoid vomiting, acidifies urine o It should not be given to pts with impaired renal or hepatic function  Other means include arginine hydrochloride, ascorbic acid and calcium chloride by mouth 72
  • 76. Marc Imhotep Cray, M.D. 76 Drugs and the Kidney and Prescribing in Renal Disease
  • 77. Marc Imhotep Cray, M.D. Drugs and the Kidney Drug-induced renal disease Drugs and other chemicals damage kidney by 3 major mechanisms: 1. Direct biochemical effect Substances that cause such toxicity include:  heavy metals, e.g. mercury, gold, iron, lead  antimicrobials, e.g. aminoglycosides, amphotericin, cephalosporins  iodinated radiological contrast media, e.g. agents for visualizing GIT  analgesics, e.g. NSAID combinations  solvents, e.g. carbon tetrachloride, ethylene glycol 2. Indirect biochemical effect:  cytotoxic drugs and uricosurics may cause urate to be precipitated in tubule  calciferol may cause renal calcification by inducing hypercalcemia  diuretic and laxative abuse can cause tubular damage secondary to K+ and Na+ depletion  anticoagulants may cause hemorrhage into kidney
  • 78. Marc Imhotep Cray, M.D. Drugs and the Kidney (2) Drug-induced renal disease cont. 3. Immunological effect  A wide range of drugs produces a wide range of injuries:  including phenytoin, gold, penicillins, hydralazine, isoniazid, rifampin, penicillamine, probenecid, sulfonamides  drugs may cause damage by more than one of above mechanisms, e.g. gold  Sites and pathological types of injury are: o Glomerular damage, eg. Penicillamine= damage from circulating immune complexes o Tubule damage, eg. acids, e.g. salicylate (aspirin), cephalosporins & bases, e.g. aminoglycosides, Heavy metals and contrast media o Tubule obstruction, eg. Methotrexate is relatively insoluble at low urine pH and can precipitate in distal nephron
  • 79. Drugs and the Kidney (3)  Other drug-induced lesions of kidney include:  Vasculitis, caused by allopurinol, isoniazid, sulfonamides  Allergic interstitial nephritis, caused by penicillins (especially), thiazides, allopurinol, phenytoin, sulfonamides  Drug-induced lupus erythematosus, caused by hydralazine, procainamide, sulfasalazine  Drugs may thus induce any of common clinical syndromes of renal injury, namely:  Acute renal failure, e.g. aminoglycosides, cisplatin  Nephrotic syndrome, e.g. penicillamine, gold, captopril (only at higher doses than now recommended)  Chronic renal failure, e.g. NSAIDs.  Functional impairment, i.e. reduced ability to dilute and concentrate urine (lithium), potassium loss in urine (loop diuretics), acid–base imbalance (acetazolamide)
  • 80. Marc Imhotep Cray, M.D. Prescribing In Renal Disease  Drugs may:  exacerbate renal disease (see previous section)  be ineffective, e.g. thiazide diuretics in moderate or severe renal failure; uricosurics  be potentiated by accumulation due to failure of renal excretion  First option is to seek an alternative drug that does not depend on renal elimination  NB: Problems of safety arise for pts. w impaired renal function who must be treated w a drug that is potentially toxic and that is wholly or largely eliminated by kidney  A knowledge of, or at least access to, sources of pharmacokinetic data is essential for safe therapy for such patients, o e.g. manufacturers’ data, formularies and specialist journals
  • 81. Marc Imhotep Cray, M.D. Prescribing In Renal Disease (2) Drug t½ (h) in normal and severely impaired renal function Glomerular filtration rate <5 mL/min (normal value is 120 mL/min). These values illustrate major effect of impaired renal function on elimination of certain drugs. Depending on circumstances, alternative drugs must be found or special care exercised when prescribing drugs that depend significantly on kidney for elimination. Modifiedafter:BennettPN,BrownMJandSharmaP.Clinical Pharmacology11thEd.Edinburgh:ChurchillLivingstone,2012.
  • 82. Marc Imhotep Cray, M.D. Prescribing In Renal Disease (3) t½ of other drugs, where activity is terminated by metabolism, is unaltered by renal impairment   but many such drugs produce pharmacologically active metabolites that are more water soluble than parent drug rely on kidney for their elimination, and accumulate in renal failure, e.g. o acebutolol, o diazepam, o warfarin, o meperidine Majority of drugs fall into an intermediate class =  partly metabolized and  partly eliminated unchanged by kidney
  • 83. Marc Imhotep Cray, M.D. Prescribing In Renal Disease (4)  Administering correct dose to a patient with renal disease must take into account both 1. extent drug normally relies on renal elimination and 2. degree of renal impairment  Best guide to renal impairment is creatinine clearance and not serum creatinine level itself as serum Cr can be notoriously misleading in elderly and at extremes of body mass  CrCl can be predicted from serum creatinine concentration, sex, age and weight using Cockcroft–Gault formula o A number of free online calculators are available, e.g. http://www.medical- calculator.nl/calculator/GFR/ To learn more see: Tutorial_3 Renal Drug Elimination and Dosing in Renal Impairment
  • 84. Marc Imhotep Cray, M.D. Prescribing In Renal Disease (5) Dose adjustment for patients with renal impairment  Adjustment of initial dose (or where necessary priming or loading dose) is generally unnecessary, as volume into which drug has to distribute should be same in uremic and healthy subject  There are exceptions to this rule of thumb for example, volume of distribution of digoxin is contracted in urinemic patients due to altered tissue binding of drug  Adjustment of maintenance dose involves either reducing each dose given or lengthening time between doses  Special caution is needed when patient is hypoproteinemia and drug is usually extensively plasma protein bound, or in advanced renal disease when accumulated metabolic products may compete for protein binding sites  Careful observation is required in early stages of dosing until response to drug can be gauged
  • 85. Prescribing In Renal Disease (5) General rules 1. Drugs that are completely or largely excreted by kidney, or drugs that produce active, renally eliminated metabolites:  give a normal or, if there is special cause for caution (as discussed above), a slightly reduced initial dose, and lower maintenance dose or lengthen dose interval in proportion to reduction in creatinine clearance 2. Drugs that are completely or largely metabolized to inactive products:  Give normal doses  When note of special caution (as above) applies, a modest reduction of initial dose and maintenance dose rate are justified while drug effects are assessed 3. Drugs that are partly eliminated by kidney and partly metabolized:  give a normal initial dose and modify maintenance dose or dose interval in light of what is known about patient’s renal function and drug its dependence on renal elimination and its inherent toxicity
  • 86. Marc Imhotep Cray, M.D. Prescribing In Renal Disease (6) Recall that time to reach steady-state blood concentration is dependent only on drug t½, and a drug reaches 97% of its ultimate steady-state concentration in 4-5 x t½  Thus, if t½ is prolonged by renal impairment, so also will be the time to reach steady state. Schemes for modifying drug dosage for patients with renal disease diminish but do not remove their increased risk of adverse effects  patients should be observed carefully throughout a course of drug TX  Where service is available, dosing should be monitored by drug plasma concentration measurements
  • 87. Marc Imhotep Cray, M.D. ADH Antagonists 87  Demeclocycline  Lithium  Lixivaptan  Satavaptan  Conivaptan  Tolvaptan
  • 88. Marc Imhotep Cray, M.D. Antidiuretic Hormone Antagonists 88  A variety of medical conditions, including  Congestive heart failure (CHF) and  Syndrome of inappropriate ADH secretion (SIADH) cause water retention as a result of excessive ADH secretion o Inability to form dilute urine in fully hydrated condition is characteristic of SIADH • Antagonists of ADH are needed to treat this condition  Patients with CHF who are on diuretics frequently develop hyponatremia secondary to excessive ADH secretion Dangerous hyponatremia can result
  • 89. Marc Imhotep Cray, M.D. Antidiuretic Hormone Antagonists (2) 89 Until recently, two nonselective agents  lithium and  demeclocycline (a tetracycline antimicrobial drug), were used for their well-known interference with ADH activity  Mechanism for this interference has not been completely determined for either of these agents  Demeclocycline is used more often than lithium because of many adverse effects of lithium administration  Demeclocycline is now being rapidly replaced by several specific ADH receptor antagonists (vaptans), which have yielded good clinical results
  • 90. Marc Imhotep Cray, M.D. Antidiuretic Hormone Antagonists (3) 90  There are 3 known vasopressin receptors, V1a , V1b , and V2  V1 receptors are expressed in vasculature and CNS  V2 receptors are expressed specifically in kidney  Conivaptan (currently available only for intravenous use) exhibits activity against both V1a and V2 receptors  Oral agents tolvaptan, lixivaptan, and satavaptan are selectively active against V2 receptor  Tolvaptan, is very effective in treatment of hyponatremia, SIADH and as an adjunct to standard diuretic therapy in patients with CHF Vaptans
  • 91. Marc Imhotep Cray, M.D. Antidiuretic Hormone Antagonists (4) 91 Pharmacokinetics  Half-life of conivaptan and demeclocycline is 5–10 hours, while that of tolvaptan is 12–24 hours Pharmacodynamics  Antidiuretic hormone antagonists inhibit effects of ADH in collecting tubule  Conivaptan and tolvaptan are direct ADH receptor antagonists  both lithium and demeclocycline reduce ADH-induced cAMP by mechanisms that are not completely yet clarified
  • 92. Marc Imhotep Cray, M.D. Antidiuretic Hormone Antagonists (5) 92 Clinical Indications & Dosage A. Syndrome of Inappropriate ADH Secretion  For SIADH, water restriction is often treatment of choice  ADH antagonists are used to manage SIADH when water restriction has failed to correct abnormality  Generally occurs in outpatient setting, where water restriction cannot be enforced, but Can occur in hospital when large quantities of intravenous fluid are needed for other purposes o Demeclocycline (600–1200 mg/d) or tolvaptan (15–60 mg/d) can be used for SIADH • Appropriate plasma levels of demeclocycline (2 mcg/mL) should be maintained by monitoring • Tolvaptan levels are not routinely monitored
  • 93. Marc Imhotep Cray, M.D. Antidiuretic Hormone Antagonists (6) 93 Clinical Indications cont. B. Other Causes of Elevated Antidiuretic Hormone  Antidiuretic hormone is also elevated in response to diminished effective circulating blood volume, as often occurs in heart failure  When Tx by volume replacement is not desirable, hyponatremia may result  As for SIADH, water restriction is often treatment of choice o In patients with heart failure, this approach is often unsuccessful in view of increased thirst and large number of oral medications being used  For patients with heart failure, intravenous conivaptan may be particularly useful b/c it has been found that blockade of V1a receptors leads to decreased peripheral vascular resistance and increased cardiac output
  • 94. Marc Imhotep Cray, M.D. Antidiuretic Hormone Antagonists (7) 94 Toxicity A. Nephrogenic Diabetes Insipidus If serum Na + is not monitored closely, any ADH antagonist can cause severe hypernatremia and nephrogenic diabetes insipidus  If lithium is being used for a psychiatric disorder, nephrogenic diabetes insipidus can be treated with a thiazide diuretic or amiloride B. Renal Failure Both lithium and demeclocycline have been reported to cause acute renal failure  Long-term lithium therapy may cause chronic interstitial nephritis
  • 95. Marc Imhotep Cray, M.D. Antidiuretic Hormone Antagonists (8) 95 C. Other Adverse Effects Dry mouth and thirst are common with many of these drugs Tolvaptan may cause hypotension Multiple adverse effects associated with lithium therapy have been found and are discussed in CNS Drugs Demeclocycline should be avoided in patients with liver disease and in children younger than 12 years
  • 96. Marc Imhotep Cray, M.D. Renal Drugs Summary Table Rosenfeld GC and Loose DS. Board Review Series Pharmacology 6th ed. Philadelphia, PA: Lippincott Williams & Wilkins, 2014.
  • 97. Marc Imhotep Cray, M.D. Case-based Discussions 97
  • 98. Marc Imhotep Cray, M.D. Case 7-Diuretics 98 A 64-year-old female with a past medical history of coronary artery disease, hypertension, and congestive heart failure (CHF) presents with dyspnea at rest and with exertion, orthopnea, and lower extremity pitting edema. Her symptoms have worsened over the last 2 weeks and also include orthopnea, worsening exercise tolerance, and tachypnea. On examination, she is notably dyspneic and tachypneic, and also has jugular venous distension, 2+pitting edema, and rales on lung examination. Patient is also found to have an audible S3. Her chest x-ray, pro-Brain Natriuretic Peptide (BNP) level, and echocardiogram confirm the clinical suspicion of CHF exacerbation with pulmonary edema. She is already on maximal medical therapy with an ACE inhibitor, beta blocker, statin, and aspirin. She is appropriately placed on oxygen and given intravenous furosemide. _ What is the mechanism of action of furosemide? _ What electrolyte abnormalities can be caused by furosemide?
  • 99. Marc Imhotep Cray, M.D. Summary: 99 A 64-year-old woman with pulmonary edema is prescribed furosemide. • Mechanism of action of furosemide: Inhibit active NaCl reabsorption in the ascending limb of the loop of Henle, increasing water and electrolyte excretion. • Potential electrolyte abnormalities: Hypokalemia, hypomagnesemia, and metabolic alkalosis because of enhanced H + excretion.
  • 100. Marc Imhotep Cray, M.D. Clinical Correlation 100  Loop diuretics given intravenously promote diuresis within minutes, making them ideal for the treatment of acute pulmonary edema.  Furosemide is the prototype and most widely used drug in this class.  Loop diuretics inhibit NaCl reabsorption in the ascending limb of the loop of Henle. This causes a marked increase in the excretion of both water and electrolytes.  The excretion of potassium, magnesium, and calcium ions are all increased, which may cause clinically significant adverse effects.  A metabolic alkalosis may also occur as a result of the excretion of hydrogen ions. o However, the ability to cause excretion of these electrolytes may also provide a clinical benefit in certain situations. o Forced diuresis by giving IV saline and furosemide is a primary method of treatment of hypercalcemia.
  • 101. Marc Imhotep Cray, M.D. Case 8-Nondiuretic Inhibitors of Tubular Transport 101 Following his third episode of gouty arthritis, a 50-year-old man sees you in the clinic. Each case was successfully treated acutely; however, your patient is interested in trying to prevent future episodes. He is not on regular medications and has a normal physical examination today. Blood work reveals an elevated serum uric acid level and otherwise normal renal function and electrolytes. A 24-hour urine collection for uric acid reveals that he is under-excreting uric acid. Suspecting that this is the cause of his recurrent gout, you place him on probenecid. _ What is the mechanism of action of probenecid? _ Which drugs could have their excretion inhibited by probenecid?
  • 102. Marc Imhotep Cray, M.D. Summary: 102 A 50-year-old man with recurrent gout is prescribed probenecid. • Mechanism of action of probenecid: Inhibits secretion of organic acids and decreases reabsorption of uric acid, causing a net increase in secretion. • Other drugs whose secretion could be inhibited: Penicillin, indomethacin, and methotrexate.
  • 103. Marc Imhotep Cray, M.D. Clinical Correlation 103  Gout is a disease in which uric acid crystals deposit in joints, causing an extremely painful acute inflammatory arthritis.  Persons with recurrent gout often have chronically elevated levels of uric acid in their blood. This hyperuricemia is frequently caused by either overproduction of uric acid or under-excretion of uric acid by the kidneys.  Probenecid (and other uricosuric drugs) promotes the excretion of uric acid. o It works by inhibiting the secretion of organic acids from the plasma into the tubular lumen and blocking the reuptake of uric acid. o The net result of this is an increase in the excretion of uric acid.  The benefit of this is the prevention of recurrent gout attacks in chronic under-excreters of uric acid.  In those individuals who overproduce uric acid, allopurinol or febuxostat is used. o These drugs inhibit xanthine oxidase, a key enzyme in the production of uric acid. o For patients with severe gout refractory to the above drugs, IV infusion of pegloticase can quickly reduce serum urate and reduce deposits in joints.
  • 104. Marc Imhotep Cray, M.D. Practice Questions & Answers/Explanations 104
  • 105. Marc Imhotep Cray, M.D. Question 1 A patient taking an oral diuretic for about 6 months presents with elevated fasting and postprandial blood glucose levels. You check the patient’s HbA1c and find it is elevated compared with normal baseline values obtained 6 months ago. You suspect the glycemic problems are diuretic-induced. What was the most likely cause? a. Acetazolamide b. Amiloride c. Chlorothiazide d. Spironolactone e. Triamterene 91
  • 106. Marc Imhotep Cray, M.D. Answer 1 The answer is c. Thiazides and thiazide-like diuretics (eg, chlorthalidone, metolazone) tend to elevate blood glucose levels, impair glucose tolerance, and cause frank hyperglycemia. Several mechanisms have been proposed to explain the effect: insulin resistance is the most likely mechanism. Elevations of blood glucose levels, or other manifestations of glycemic control, are rarely associated with treatment with acetazolamide (a), amiloride (b), spironolactone (d), or triamterene (e). Refs. G&G, pp 686-690; Katzung, pp 260-261. NB: You may recall that diazoxide (mainly used as a parenteral drug for prompt lowering of blood pressure) can be used in its oral dosage form to raise blood glucose levels in some hypoglycemic states. It is, chemically, a thiazide, but is not used as a diuretic.) 92
  • 107. Question 2 A patient with essential hypertension is being treated with hydrochlorothiazide and a calcium channel blocker, and is doing well. He also takes atorvastatin for hypercholesterolemia, and aspirin to reduce his risk of an acute coronary syndrome. He is now diagnosed with a seizure disorder. We begin therapy with one of the suitable anticonvulsants that, fortunately, does not alter the metabolism of any of the medications prescribed for his cardiovascular problems. We’ve also learn that systemic administration of acetazolamide may prove to be a useful adjunct to the anticonvulsant therapy: the metabolic acidosis it causes may help suppress seizure development or spread. So, we start acetazolamide therapy too. What is the most likely outcome of adding the acetazolamide? a. Excessive rises of plasma sodium concentration b. Hypertensive crisis (antagonism of both antihypertensive drugs) c. Hypokalemia via synergistic actions with the thiazide d. Spontaneous bleeding (potentiation of aspirin’s actions) e. Sudden circulating volume expansion, onset of heart failure 93
  • 108. Marc Imhotep Cray, M.D. Answer 2 The answer is c. We seldom administer acetazolamide as a diuretic, because its effects are “mild”; associated with significant changes of both urine pH (up) and blood pH (down; metabolic acidosis); and self-limiting (once sufficient bicarbonate has been lost from the blood, into the urine, refractoriness to further diuresis occurs). More often we administer acetazolamide and other carbonic anhydrase inhibitors for nonrenal/noncardiovascular problems, such as to lower intraocular pressure in some cases of glaucoma (carbonic anhydrase inhibitors inhibit aqueous humor formation) or as an adjunct to anticonvulsant therapy as described here. As a result, we may forget that these systemically administered drugs are diuretics, one common property of all the diuretics being increased renal sodium loss (a natriuretic effect; thus, answer a is not correct). We may even forget that carbonic anhydrase inhibitors, given systemically, are potassium-wasting diuretics: they act proximally and deliver extra sodium distally where, at the principal cells of the nephron, some extra Na+ is taken up in exchange for additional K+ that gets eliminated in the urine. 94
  • 109. Answer 2 cont. In this scenario the patient is taking a thiazide, which is obviously potassium-wasting and has potential in its own right to cause hypokalemia. Add a carbonic anhydrase to the regimen and the risks of hypokalemia increase. Acetazolamide does not antagonize the antihypertensive effects of thiazides or calcium channel blockers, nor provoke hypertension or a hypertensive crisis (b). If there were any interactions between the acetazolamide and the aspirin, it would be antagonism, not potentiation (d) of aspirin’s antiplatelet effects. Aspirin undergoes renal tubular reabsorption, and that is a pH-dependent effect. Aspirin’s reabsorption is reduced (that is, its excretion increases) in an alkaline urine, which is precisely what occurs with acetazolamide. (You should recall that alkalinizing the urine is an important adjunctive measure in treating severe salicylate poisoning, in part because it reduces tubular reabsorption of salicylate.) There is no reason to suspect sudden rises of blood volume, with or without concomitant heart failure from that (e). Indeed, the added diuresis from the acetazolamide may, at least transiently, potentiate the effects of the thiazide on urine volume, blood pressure, or both. Refs. G&G, pp 677-681; Katzung, pp 256-257, 261-262, 265. 95
  • 110. Marc Imhotep Cray, M.D. Question 3 An elderly patient with a history of heart disease is brought to the emergency room with difficulty breathing. Examination reveals that she has pulmonary edema. Which treatment is indicated? A. Acetazolamide. B. Chlorthalidone. C. Furosemide. D. Hydrochlorothiazide. E. Spironolactone. 96
  • 111. Marc Imhotep Cray, M.D. Answer 3 Correct answer is C. This is a potentially fatal situation. It is important to administer a diuretic that will reduce fluid accumulation in the lungs and, thus, improve oxygenation and heart function. The loop diuretics are most effective in removing large fluid volumes from the body and are the treatment of choice in this situation. In this situation, furosemide should be administered intravenously. The other choices are inappropriate. 97
  • 112. Marc Imhotep Cray, M.D. Question 4 A group of college students is planning a mountain climbing trip to the Andes. Which would be appropriate for them to take to prevent mountain sickness? A. A thiazide diuretic such as hydrochlorothiazide. B. An anticholinergic such as atropine. C. A carbonic anhydrase inhibitor such as acetazolamide. D. A loop diuretic such as furosemide. E. A β-blocker such as metoprolol. 98
  • 113. Marc Imhotep Cray, M.D. Answer 4 Correct answer is C. Acetazolamide is used prophylactically for several days before an ascent above 10,000 feet. This treatment prevents the cerebral and pulmonary problems associated with the syndrome as well as other difficulties, such as nausea. 99
  • 114. Marc Imhotep Cray, M.D. Question 5 An alcoholic male has developed hepatic cirrhosis. To control the ascites and edema, which should be prescribed? A. Acetazolamide. B. Chlorthalidone. C. Furosemide. D. Hydrochlorothiazide. E. Spironolactone. 100
  • 115. Marc Imhotep Cray, M.D. Answer 5 Correct answer is E. Spironolactone is very effective in the treatment of hepatic edema. These patients are frequently resistant to the diuretic action of loop diuretics, although a combination with spironolactone may be beneficial. The other agents are not indicated. 101
  • 116. Marc Imhotep Cray, M.D. Question 6 A 55-year-old male with kidney stones has been placed on a diuretic to decrease calcium excretion. However, after a few weeks, he develops an attack of gout. Which diuretic was he taking? A. Furosemide. B. Hydrochlorothiazide. C. Spironolactone. D. Triamterene. E. Urea. 102
  • 117. Marc Imhotep Cray, M.D. Answer 6 Correct answer is B. Hydrochlorothiazide is effective in increasing calcium reabsorption, thus decreasing the amount of calcium excreted, and decreasing the formation of kidney stones that contain calcium phosphate or calcium oxalate. However, hydrochlorothiazide can also inhibit the excretion of uric acid and cause its accumulation, leading to an attack of gout in some individuals. Furosemide increases the excretion of calcium, whereas the K+-sparing osmotic diuretics, spironolactone and triamterene, and urea do not have an effect. 103
  • 118. Marc Imhotep Cray, M.D. Question 7 A 75-year-old woman with hypertension is being treated with a thiazide. Her blood pressure responds well and reads at 120/76 mm Hg. After several months on the medication, she complains of being tired and weak. An analysis of the blood indicates low values for which of the following? A. Calcium. B. Glucose. C. Potassium. D. Sodium. E. Uric acid. 104
  • 119. Marc Imhotep Cray, M.D. Answer 7 Correct answer is C. Hypokalemia is a common adverse effect of the thiazides and causes fatigue and lethargy in the patient. Supplementation with potassium chloride or foods high in K+ corrects the problem. Alternatively, a potassium sparing diuretic, such as spironolactone, may be added. Calcium, uric acid, and glucose are usually elevated by thiazide diuretics. Sodium loss would not weaken the patient. 105
  • 120. Marc Imhotep Cray, M.D. Question 8 Which is contraindicated in a patient with hyperkalemia? A. Acetazolamide. B. Chlorthalidone. C. Chlorothiazide. D. Ethacrynic acid. E. Spironolactone. 106
  • 121. Marc Imhotep Cray, M.D. Answer 8 Correct answer is E. Spironolactone acts in the collecting tubule to inhibit Na+ reabsorption and K+ excretion. It is extremely important that patients who are treated with any potassium-sparing diuretic be closely monitored for potassium levels. Exogenous potassium supplementation is usually discontinued when potassium-sparing diuretic therapy is instituted and spironolactone is contraindicated in patients with hyperkalemia. The other drugs promote the excretion of potassium. 107
  • 122. Marc Imhotep Cray, M.D. Question 9 Which of the following should be avoided in a patient with a history of severe anaphylactic reaction to sulfa medications? A. Amiloride. B. Hydrochlorothiazide. C. Mannitol. D. Spironolactone. E. Triamterene. 108
  • 123. Marc Imhotep Cray, M.D. Answer 9 Correct answer is B. Hydrochlorothiazide, like many thiazide and thiazide-like diuretics, contains a sulfa moiety within its chemical structure. It is important to avoid use in those individuals with severe hypersensitivity to sulfa medications. It may be used with caution, however, in those with only minor reaction to sulfa medications. 109
  • 124. Marc Imhotep Cray, M.D. Question 10 A male patient is placed on a new medication and notes that his breasts have become enlarged and tender to the touch. Which medication is he most likely taking? A. Chlorthalidone. B. Furosemide. C. Hydrochlorothiazide. D. Spironolactone. E. Triamterene. 110
  • 125. Marc Imhotep Cray, M.D. Answer 10 Correct answer = D. An adverse drug reaction to spironolactone is gynecomastia due to its effects on androgens and progesterone in the body. Eplerenone may be a suitable alternative if the patient is in need of an aldosterone antagonist but has a history of gynecomastia. 111
  • 126. Marc Imhotep Cray, M.D. Question 11 A patient presents to the emergency department with an extreme headache. After a thorough workup, the attending physician concludes that the pain is due to increased intracranial pressure. Which diuretic would work best to reduce this pressure? A. Acetazolamide. B. Indapamide. C. Furosemide. D. Hydrochlorothiazide. E. Mannitol. 112
  • 127. Marc Imhotep Cray, M.D. Answer 11 Correct answer = E. Osmotic diuretics, such as mannitol, are a mainstay of treatment for patients with increased intracranial pressure or acute renal failure due to shock, drug toxicities, and trauma. 113
  • 128. Marc Imhotep Cray, M.D. Question 12 Which diuretic has been shown to improve blood pressure in resistant hypertension or those already treated with three blood pressure medications including a thiazide or thiazide-like diuretic? A. Chlorthalidone. B. Indapamide. C. Furosemide. D. Mannitol. E. Spironolactone. 114
  • 129. Marc Imhotep Cray, M.D. Answer 12 Correct answer = E. Resistant hypertension, defined by the use of three or more medications without reaching the blood pressure goal, often responds well to aldosterone antagonists. This effect can be seen in those with or without elevated aldosterone levels. 115
  • 130. Marc Imhotep Cray, M.D. See next slide for sources and links to additional study tools and resources. 130
  • 131. Marc Imhotep Cray, M.D. Sources and further study: eLearning Renal cloud folder tools and resources MedPharm Guidebook: Unit 9 Drugs Used to Affect Renal Function Renal Pharmacology eNotes Clinical Pharmacology Cases 7, 8, & 55 (Learning Triggers) Textbooks Brunton LL, Chabner BA , Knollmann BC (Eds.). Goodman and Gilman’s The Pharmacological Basis of Therapeutics. 12th ed. New York: McGraw-Hill, 2011 Katzung, Masters, Trevor. Basic and Clinical Pharmacology, 12th ed. New York: McGraw-Hill, 2012 Mulroney SE. and Myers AK. Netter's Essential Physiology. Philadelphia: Saunders, 2009 Raff RB, Rawls SM, Beyzarov EP. Netter's Illustrated Pharmacology, Updated Edition. Philadelphia: Sanders, 2014 Toy E C. et.al. Case Files-Pharmacology Lange 3rd ed. New York: McGraw-Hill 2014. 131

Editor's Notes

  1. Glomerulus in a human kidney (scanning electron micrograph 5003.) From: Widmaier EP, Raff H & Strang KT Vander’s Human Physiology: The Mechanisms Of Body Function, 13th Ed. New York, NY: McGraw-Hill Companies, Inc., 2014: 490
  2. Photograph of a longitudinal section of a human kidney and ureter, Renal cell carcinoma, Acute Poststreptococcal glomerulonephritis IF—(“starry sky”) granular appearance (“lumpy-bumpy”) due to IgG, IgM, and C3 deposition along GBM and mesangium, Cross-section of glomerulus, polycystic kidney disease.
  3. thiazide-sensitive Na–Cl co-transporter
  4. TAL, thick ascending limb of the loop of Henle The ‘hairpin’ structure of the loop thus confers on it the property of a countercurrent multiplier, i.e. by active transport of ions a small change in osmolality laterally across the tubular epithelium is converted into a steep vertical osmotic gradient.
  5. ROMK, rectifying outer medullary potassium channel
  6. Sites of action of diuretic drugs. Inset cartoons show the transporters and ion channels targeted in tubular cells at these sites. ENaC, epithelial sodium channel; NCCT, thiazide-sensitive Na–Cl co-transporter; NKCC2, Na–K–2Cl co-transporter; ROMK, rectifying outer medullary potassium channel.
  7. Sulfa containing drugs: Sulfonamide antibiotics, Sulfasalazine, Probenecid, Furosemide, Acetazolamide, Celecoxib, Thiazides, Sulfonylureas.
  8. Characterized by fever, bullae formation and necrosis, sloughing of skin at dermal-epidermal junction, high mortality rate. Typically 2 mucous membranes are involved G H (Around Eyes/Mouth), and targetoid skin lesions may appear, as seen in erythema multiforme. Usually associated with adverse drug reaction. A more severe form of Stevens-Johnson syndrome (SJS) with > 30% of the body surface area involved is toxic epidermal necrolysis I J (TEN). 10–30% involvement denotes SJS-TEN.
  9. With the exception of metolazone, thiazides are not effective when renal function is moderately impaired (GFR <20 mL/min), because they are not filtered in sufficient concentration to inhibit the NCC.
  10. epithelial sodium channel
  11. Methylxanthines methylxanthines (theophylline, caffeine) are discussed elsewhere Their mild diuretic action depends in part on smooth muscle relaxation in the afferent arteriolar bed increasing renal blood flow, and in part on a direct inhibitory effect on salt reabsorption in the proximal tubule. Their uses in medicine depend on other properties.
  12. (thiazides actually raise intraocular pressure slightly)
  13. Hypokalemia predisposes pts Tx with cardiac glycosides (digoxin) to toxicity (permissive for digoxin binding at K+-binding site on Na+/K+ ATPase) bradyarrhythmia, heart block The consequence of having abnormal serum K is that it can lead to arrhythmias and even complete heart block. Notice the words in the previous sentence (“abnormal” serum K rather than “increased” or “decreased” serum K). The reason for this is that intuitively, one would expect serum K to rise once the Na/K pump is blocked, and indeed this is what happens in acute cases of overdose. However, in practice, patients are rarely taking digitalis alone because it is not a drug of first resort. Patients will typically also be taking thiazide or loop diuretics, both of which expel K from the body through the renal system. Therefore, in chronic cases of digitalis toxicity, it is hypokalemia (rather than hyperkalemia) that causes cardiac dysrhythmias. This is why one should consider either potassium-sparing diuretics or potassium supplements in patients on long-term glycoside regimens.
  14. Lower doses required compared to edematous conditions
  15. These may not be obvious, for example, to patient who is using an unprescribed ‘low sodium’ salt substitute to reduce their salt (NaCl) intake
  16. cardiac manifestations bradyarrhythmia, heart block and hyperkalemia, both of which suggest digitalis toxicity).
  17. NB indinavir requires acidification
  18. methylenedioxymethamphetamine
  19. Tubule damage. By concentrating 180 L glomerular filtrate into 1.5 L urine each day, renal tubule cells are exposed to much greater amounts of solutes and environmental toxins than are other cells in the body. The proximal tubule, through Which most water is reabsorbed, experiences the greatest concentration and so suffers most drug-induced injury.
  20. The profound influence of impaired renal function on the elimination of some drugs is illustrated in Table 27.1.
  21. A number of free online calculators are available, e.g. http://www.medical-calculator.nl/ calculator/GFR/
  22. Recall that time to reach steady-state blood concentration is dependent only on drug t½, and a drug reaches 97% of its ultimate steady-state concentration in 4-5 x t½. Thus, if t½ is prolonged by renal impairment, so also will be the time to reach steady state. Schemes for modifying drug dosage for patients with renal disease diminish but do not remove their increased risk of adverse effects; such patients should be observed particularly carefully throughout a course of drug therapy. Where the service is available, dosing should be monitored by drug plasma concentration measurements.
  23. Conivaptan and tolvaptan are ADH antagonists. Demeclocycline was previously used for this purpose. Lithium also has ADH-antagonist effects but is never used for this purpose. ADH facilitates water reabsorption from the collecting tubule by activation of V2 receptors, which stimulate adenylyl cyclase via Gs. The increased cyclic adenosine monophosphate (cAMP) causes the insertion of additional aquaporin AQP2 water channels into the luminal membrane in this part of the tubule (Figure 15–6). Conivaptan is an ADH inhibitor at V1a and V2 receptors. Tolvaptan is a more selective V2 blocker with little V1 affinity. Demeclocycline and lithium inhibit the action of ADH at some point distal to the generation of cAMP and presumably interfere with the insertion of water channels into the membrane.
  24. Antagonists—ADH antagonists oppose the actions of ADH and other naturally occurring peptides that act on the same V2 receptor. Such peptides are produced by certain tumors (eg, small cell carcinoma of the lung) and can cause significant water retention and dangerous hyponatremia. This syndrome of inappropriate ADH secretion (SIADH) can be treated with demeclocycline and conivaptan. Lithium also works but has greater toxicity.
  25. Diuretics A 64-year-old woman with pulmonary edema is prescribed furosemide. • Mechanism of action of furosemide: Inhibit active NaCl reabsorption in the ascending limb of the loop of Henle, increasing water and electrolyte excretion. • Potential electrolyte abnormalities: Hypokalemia, hypomagnesemia, and metabolic alkalosis because of enhanced H + excretion.
  26. Nondiuretic Inhibitors of Tubular Transport A 50-year-old man with recurrent gout is prescribed probenecid. • Mechanism of action of probenecid: Inhibits secretion of organic acids and decreases reabsorption of uric acid, causing a net increase in secretion. • Other drugs whose secretion could be inhibited: Penicillin, indomethacin, and methotrexate.
  27. You might recall that diazoxide [mainly used as a parenteral drug for prompt lowering of blood pressure] can be used in its oral dosage form to raise blood glucose levels in some hypoglycemic states. It is, chemically, a thiazide, but is not used as a diuretic.)